为了正常的体验网站,请在浏览器设置里面开启Javascript功能!

学生用 英语阅读理解四大题型专讲专练

2017-12-20 50页 doc 314KB 57阅读

用户头像

is_589748

暂无简介

举报
学生用 英语阅读理解四大题型专讲专练学生用 英语阅读理解四大题型专讲专练 阅读理解题型专讲专练 【细节理解题】 做细节理解题时,大多数学生易出现的问题就是阅读速度太慢,缺乏一定的快速阅读技巧,考生要培养自己快速获取信息的能力。解答此类试题时,不必通篇细看原文,而应采取“带着问题找答案”的方法,先从问题中抓住关键性词语,然后以此为线索,运用略读及查读的技巧快速在文章中寻找与此问题相关的段落、语句,仔细品味,对照比较,确定答案。除了运用扫读法外,还可以兼用排除法,将“无此细节”和“与此细节相反”的选项排除。要快速地辨认和记忆事实或细节,就需要恰当地使用查阅的方...
学生用  英语阅读理解四大题型专讲专练
学生用 英语阅读理解四大型专讲专练 阅读理解题型专讲专练 【细节理解题】 做细节理解题时,大多数学生易出现的问题就是阅读速度太慢,缺乏一定的快速阅读技巧,考生要培养自己快速获取信息的能力。解答此类试题时,不必通篇细看原文,而应采取“带着问题找答案”的方法,先从问题中抓住关键性词语,然后以此为线索,运用略读及查读的技巧快速在文章中寻找与此问题相关的段落、语句,仔细品味,对照比较,确定答案。除了运用扫读法外,还可以兼用排除法,将“无此细节”和“与此细节相反”的选项排除。要快速地辨认和记忆事实或细节,就需要恰当地使用查阅的方法及技巧。查阅是在读者对材料有所熟悉的情况下进行的,它的特点是带着问题去寻求答案,它往往与略读综合使用。 〖第一招〗 直接细节理解题答案与原文挂钩,在文中可直接找到答案,但往往与原文中的语句并非一模一样,而是用不同的词语或句型结构表达相同的意思。间接细节理解题需要通过有关词语和的转换,利用主要事实、图表、图形来获取信息,然后利用因果、类比、时间、空间等关系将零碎的细节经过一系列加工、整理,方能做出正确的判断,此类试题在高考中占大多数。其常见命题方式有: 1、特殊疑问句形式。以what, who, when, where, which, how much / many 等引出的问题; 2、是非题。通常以true / false, not true / false 提问以及以according to the text 开头; 3、填空题。通常涉及与主题有关的事实和细节; 4、就文中数字、排序、识图等提问。 〖第二招〗 略读材料,大概了解全文,掌握其中心或主旨。 〖第三招〗 按文章的体裁,如记叙文、说明文和议论文等及作者写作的组织模式和有关的信息词,如for example, first, second 等预测应该到何处寻求自己所需要的事实。段落的组织形式常见的有时间型段落、空间型段落、列举型段落、例证型段落、程序型段落和对比型段落等。如时间型段落和空间型段落要凭借表达时间和空间的信息词按时间和空间的组织形式进行查读,寻找有关细节。 〖第四招〗 将精力放在寻找你所需要的细节上,快速通篇跳读,眼睛自左至右,自上而下呈Z型扫读,直到找到含细节句子时,就要放慢速度,仔细核对、比较,直至找到答案。 〖第五招〗 了解细节题干扰选项的特点也能有助于提高答题的正确率。一般情况下,干扰项有以下五个特点:?是原文信息,但不是题目要求的内容;?符合常识,但不是文内容;?与原句的内容极为相似,只是在程度上有些变动;?在意思上与原文大相径庭甚至完全相反;?部分正确,部分错误。值得一提的是,有时原文中的信息可能只是一个短语甚至一个单词,因此需要我们阅读中格外仔细才能捕捉到真正有用的信息。 【试题分析】 1、直接信息理解题细节的直接辨认不要求读者对客观的事实作出解释或判断,只要求从阅读材料中直接获取信息。同时还要求读者记住重要的细节,在必要的时候(做判断、推论或结论的时候)能准确而迅速地将他们回忆起来。解此类题要求考生快速抓住关键信息,直接得出答案。有时需要词句意的转换理解,将获得的信息用同义或近义的形式复述出来。NMET了许多这样的同义转换理解题,具体的要求是为阅读材料中某些词汇、短语及句型找到正确的释义。 例如: The World Trade Organization (WTO), founded on January 1, 1995, aims to encourage international trade to flow as possible, making sure that trade agreements are respected and that any disputes can be settled. In the five years since its founding, the WTO has become well known as one of the world’s most powerful economic organizations, taking its place alongside the World Bank and International Money Fund. The system of global rules for international trade, however, dates back half a century to 1948 when the General Agreement on Tariffs and Trade was formed after World War II. As time went by, is became clear that the GATT had two major drawbacks-the limited areas of trade it covered, and the lack of an effective system to settle disputes. After seven years of trade talks ending in 1994, the so-called Uruguay Round finally give birth to the WTO, complete with an effective system to settle disputes and new rules covering trade in services and intellectual property. Even after seven years of talks and 22, 500 pages of agreements, there were still problems, especially the difficulty to deal with areas of agriculture and services, which the member nations agreed to revise in 2000, The WTO, with its head office in Geneva, has 135 members with 30 more waiting to join. 总述:本文主要介绍了世贸组织的演变过程,即由最初的关贸总协定,到后来的乌拉 圭回合谈判,到1995年1月1日成立的世贸组织,其演变经历了大半个世纪,使其成为世 界上最大的国际经济组织。 45. From the passage we know that the GATT stopped working ________. A. soon after World War II ended B. a little more than 50 years after World War II C. just in the year 1994 D. seven years before the Uruguay Round talk 46. Compared with the GATT, the WTO _______. A. didn’t pay enough attention to services and intellectual property B. got its members to sign the agreements more easily C. has got to many areas of international trade to deal with to work effectively D. can do better to settle disputes in more areas of international trade 47. In the new century the WTO will ________. A. take the place of the World Bank and International Monetary Fund B. have more members and settle more problems C. make complete new rules in every area of international trade D. have new rules covering trade in services and intellectual property 【猜测词义题】 〖第一招〗利用上下文语境线索 上下文线索猜测词义 任何一篇文章中的句子在内容上都不是绝对孤立的,都跟句子所在的段落及整篇文章有关。 利用上下文提供的情景和线索,进行合乎逻辑的综合分析而推测词义,是阅读过程中的一大 关键,也是高考的热点。 如: If he thinks he can invite me out, he is all wet. I don’t like to be with him. A. drunk B. sweating C. happy D. mistaken 练习: (1)A deaf and dump guy went into a hardware store to ask for some nails. (2)The climate of the west coast is the most moderate in Canada, summers are cool and fairly dry and winters are mild, cloudy and wet. Even in mid-winter, the temperature is usually above freezing. (3)All the houses in the city collapsed during the earthquake. (4)For people who live within a stone’s throw from the office to be late to work is unforgivable. (5)The conflagration was so fierce that with just a few second one could see towering flames where the house had stood and the smoke which filled the sky could be seen for miles and miles. (6). Charlotte’s tale was inspired by the girls’ coin collection. “We’ve collected foreign coins for years-since our families went on holiday to Tenerife,” she explains. “That was before the Euro, so we put pesetas in.” The underlined wore “pesetas” in Paragraph 2 is a kind of _______. A. story B. collection C. inspiration D. foreign coin (7). In 1963 a schoolboy called Andrew Wiles reading in his school library came across the thworld’s greatest mathematical 17 century, the theorem had baffled and beaten the finest mathematical minds, including a French woman scientist who made a major advance in working out the problem, and who had to dress like a man in order to be able to study at the Ecole Polytechnique. Which of the following best explains the meaning of the word “baffle” as it is used in the text? A. To encourage people to raise questions. B. To cause difficulty in understanding. C. To provide a person with an explanation. D. To limit people’s imagination. (8). Today when many tend to worry more about their own happiness, Houng’s deeds remind us of what we usually neglect: love and care for others. Without these, none of us could survive. Houng turned down donations from others. He said he felt encouraged by kind offers, but he could depend on his own work. 42. The underlined word “donation” in Paragraph 3 probably means ______. A. invitation to give a speech B. something, especially given to help others C. questions asked by interviewers D. chances to be a hero 〖第二招〗利用定义和解释性线索 根据定义或解释猜测词义 阅读文章中的有些生词尤其是新闻报导及科普类文章中的生词,往往在其后有对该词 进行解释说明性的短语或句子,如to be, that is , mean, stand for, namely, to refer to, to mean ,in other words等,有时也以同位语、定语从句的形式出现,或用破折号、括号来表示。 1. 直接定义:如果生词是句子或段落所解释的定义,理解句子或段落本身就是推断词义。 如: In slang the term jam constitutes a state of being in which a person finds himself or herself in a difficult situation. 根据上下文的定义可知jam一词在俚语中的意思是 “困境”。 定义句的谓语 动词多为:be, mean, deal with, be considered, to be, refer to, be called, be known as, define, represent等。 2. 同位语:构成同位关系的两部分之间多用逗号连接,有时也使用破折号、冒号、分号、 引号和括号等。需要注意的是:同位语前还常有or, similarly, that is to say, in other words, namely, or other, say, i(e. 等副词或短语出现。 3. 定语从句:定语从句有时起着解释和说明的作用,据此我们可以推断出所修饰词的含义。 如: The herdsman,_who looks after sheep, earns about 650 yuan a year. 定语从句中 looks after sheep 就表明了 herdsman 的词义为“牧羊人”。 (2010年四川师大附中高三模拟题) You should never put a cotton swab or other object into the ear canal. But you can use a swab or cloth to clean the outer part of the ear. The experts agree with the old saying that you should never put anything smaller than your elbow in your ear. ( )What does a “swab” mostly probably mean? A. something soft, small and used in clinics B. something hard, long and used at home C. something thin, wet and like an elbow D. something safe, big and like a toy 如: Some ships carried cargo such as coal, oil and military supplies(军用物资),while others carried only passengers. 练习 (1)Yet, shopkeepers may have to spend extra hours to deal with problems, such as shoplifter, who always take away things from the shop without paying for them. (2)The best football, basketball and tennis players can become professional that means they will have a career in sports and will get money when they play. (3)Kleptomania is an illness of the mind that gives a person the desire to steal. (4)The word “lefty” means a person who uses his or her left hand for writing, eating and other jobs. (5)Doctors recommended that everyone exercise every day, particular those who spend many hours doing sedentary activities like reading, typing or sewing. (6)A person who has the SARS (非典型肺炎)may have symptoms like coughing and a high temperature. (7)(2011?湖南卷C篇) …The scientists used detailed genetic analysis to prove that the African savanna elephant and the African forest elephant have been distinct species for several million years. The divergence of the two species took place around the time of the divergence of Asian elephants and woolly mammoths. This result amazed all the scientists. 67. The underlined word “divergence” in paragraph 4 means “______”. A. evolution B. exhibition C. separation D. examination (7)1. The elder learn to master the Internet and to overcome what Lansdale calls the maladies of the institutionalized: loneliness, helplessness, boredom, and lose of memory. (8). Some ships carried cargo such as coal, oil and military supplies (军用物资),while others carried only passengers. (9). Scientists grow large quantities of common mould (霉菌)so that they can get penicillin from it in order to make antibiotics, that is , substances that kill germs. (10). Marine biology, the study of oceanic plants and animals and their relation, has furthered the efficient development of fisheries. (11). ,Yet, shopkeepers may have to spend extra hours to deal with problems, such as shoplifter, who always take away things from the shop without paying for them. 〖第三招〗利用因果关系 在句子或段落中,若两个事物、现象之间构成因果关系,我们可以根据这种逻辑关系推 知生词词义。如: Tom is considered an autocratic administrator because he makes decisions without seeking the opinions of others. 根据原因状语从句的内容,我们可以推断出生词autocratic指“独断专行的”。 因果关系的语境,通常由because, so ,therefore, so that, so / such… that… 等连词体现。 1. Answer the following questions by using the information taken from a dictionary page. jazz: 1. n. a type of music that originated in New Orleans and is characterized by rhythmic beats. 2. n. popular dance music influenced by jazz. 3. n. slang. Empty talk. 4. adj. of or like jazz; a jazz band, jazz records. What does the word “jazz” mean in the following sentence? Don’t give me that jazz, for I am a practical person. A. Rhythmic beats. B. A type of music. C. A kind of dance. D. Meaningless talk. 2. The biggest power failure happened yesterday. All of our ice cream and frozen foods melted. 3. He ran downstairs through the smoke-filled house to push and pull at Karen and Todd until they sat up. Then he helped each other out of the house to the safety of the garden. There, his sister and brother, taking short and quick breaths and coughing, collapsed on the lawn. 〖第四招〗利用反义词或反义关系 有的文章作者为了增强表达效果,会用一对反义词揭示事物的不同点,形成鲜明的对比,这 时只要把握其中的一词,就不难推出另一词的含义,这种句子多见unlike, although, but, yet, while, on the contrary, on the other hand, for one thing, for another, intead of, rather than等信 息词。如: Andrew is one of the most supercilious men I know. His brother, in contrast, is quite humble and modest. 该例中supercilious对许多人来说可能是个生词,但是句中短语in contrast(相对照 的,相对比的)提示我们supercilious和后面词组humble and modest(谦卑又谦虚)是对比关系。 分析出这种关系后,我们便能猜出supercilious意为“目空一切的,傲慢的”。 再如: One idea about business is that it can be treated as a game of perfect information. Quite the reverse, business polite, life itself is games which we must normally play with very imperfect information. A. Quite right. B. Time enough. C. Most unlike. D. Just the opposite. 1. Most of us agreed; however, Gary dissented. 2. He is homely, not at all as handsome as his brother. 3. “Unlike vitamin C, leadership skills can’t be easily swallowed down. They must be carefully cultivated.” The underlined word “cultivated”(Paragraph 1) roughly means ________. A. encouraged B. compared C. examined D. developed 4( One idea about business is that it can be treated as a game of perfect information. Quite the reverse , business polite, life itself is games which we must normally play with very imperfect information. Which of the following can be used in place of “quite the reverse”? A. Quite right. B. Time enough. C. Most unlikely D. Just the opposite. 5. Thousands of people got stuck in lifts. But no one panicked, We passed the time telling stories. 6. Unlike the Unite States where many different nationalities make up the population, Japan’s population is quite homogeneous. 〖第五招〗利用同义线索和同等关系 同义词替换可以为我们推测词义提供明显的语境线索。一些常见的引出同义词的标志性词语 有similarly, like, just as, also, as well等。如: Green loves to talk, and his brothers are similarly loquacious. 该句中副词similarly表明短语loves to talk与生词loquacious之间为类比关系。以此可以推 断出loquacious词义为“健谈的”。同等关系是指一个词、一组词或短语在句中作同一成分, 而且他们的词义属一范畴。只要我们认识其中一个或几个词或短语,即可确定同等关系中生 词的词性,作用和大概意思。在句子或段落中,我们可以利用熟悉的词语,根据语言环境推 断生词词义。如: Although he often had the opportunity, Mr. Tritt was never able to steal money from a customer. This would have endangered his position at the bank, and he did not want to jeopardize his future. 作者为避免重复使用endanger一词用其同义词jeopardize来替代它,由此推知 jeopardize词义为“使……陷入危险,危及、危害”。 1. 我们可以根据上下文与生词密切相关的句子,对生词进行合情合理同时合乎上下文语境 的推测。我们也可以根据生词后举出的有关例子进行适当归纳,猜测生词的词义。例如: You can take any of the periodicals:_The World of English, Foreign Language Teaching in Schools, or English Learning. 根据下文举出的英语杂志名称可知,periodicals是“期刊”之意。 根据上下文及生活常识猜测词义 2. 有时仅靠分析篇章内在逻辑关系无法猜出词义。这时,就需要运用生活经验和普通常识 确定词义。例如: The snake slithered through the grass. 根据有关蛇的生活习性的知识,我们可以推断出slither的词义为“爬行”。 (2010年湖南省六校联考) …But sometimes I just feel loath to talk to these people. Sometimes, I just want to be alone, quietly, without being forced to listen to their gossip or other useless words. ( )The underlined word “loath” in this paragraph has the closest meaning to ______. A. unwilling B. eager C. pleased D. hurt 练习 1. Millions of animals die each year on US roads, the Federal Highway administration reports. In fact, only about 80 ocelots, an endangered wild cat exist in the US today. The main reason? Roadkill. 2. mansion, church, battle site, theatre and other public halls can be preserved. 3. We should, therefore, learn to choose our words carefully and use them accurately, of they will make our speech silly and vulgar. 〖第六招〗利用例证性线索 某些冷僻的词汇后会举一个例子,使词汇具体易懂。Such as, like, for example, for instance等连接性词语往往用来列举说明前面较难理解的名词。 1. You can take any of the periodicals: The World of English, Foreign Language Teaching in School, of English learning. 2. Many United nations employees are polyglots: Mr. Simpson, for example, speaks five languages fluently. 〖第七招〗根据构词法 阅读中常会遇到一些由所熟悉的单词派生或合成的新词,可利用构词法知识来推测其意思。 《教学大纲》已经明确地将构词法列在语法附表中,因此利用所学构词法生成的词不应被认 定为生词。英语单词的构成方法主要有三种,即复合法、派生法和转化法。英语单词大多是 由词根、词头(前缀)、词尾(后缀)所组成。词根是单词最基本的部分,表达单词的基本含义。 在词根前头或后头加上前缀或后缀,可以用来引申或转变原词的意义。只要我们掌握了各种 词根、词头、词尾的基本含义,那么就可以很容易地猜测所构成的新词的含义了。中学英语 中常见的前缀和后缀有: 根据构词法来猜测词义 super,(超), inter,(在……之间), ,able(能……的), mini,(极少的,微型的),micro,(极微小的), re,(再, 反复), sub,(分支的,底下), co,(共同), post,(后), pre,(前), trans,(超越;转换), under,(在……之下; 低于……;不足), ,hood(状态;性质;时期), ,ish(如……的;有点儿……的); ,proof(防……的;…… 不能穿透的), ,scope(景), ,ship(身份;资格;权力;性质), ,some(易于……的), ,wards(向), mis,(误; 恶), un,(不;非), in,(不;非), im,(不; 非), dis,(不), non, (不;非),,less(不;无), anti,(反;防), sino,(中国)。如: He fell into a ditch and lay there, semiconscious,_for a few minutes. 根据词根conscious(清醒的,有意识的),结合前缀semi(半,部分的,不完全的), 我 们便可猜出semiconscious词义“半清醒的,半昏迷的”。 (2010年广东华南师大附中高三综合测试) Sharks are among the few animals in the world that almost never get cancer and eye cataracts. Understanding why can help us improve human health. Chemicals extracted from shark cartilage have killed cancerous tumors in laboratory animals, a research that someday could help prolong your life. ( )The underlined word “prolong” in the paragraph means ______. A. save B. protect C. lengthen D. improve 代词that/it/they/them所指代的内容多在这一句的前、后句中(特别是前句),或者前几个句 子中,找到指代的内容后把它放在那个代词的位置上,看看这句话是否合理,与前面的内容 是否一致,然后再判断它是否为正确答案。 判断代词的确指对象 1. Exhibition officials said that a person bitten by one of these snakes would need at least 80 ml of an anti-poison medicine to be saved. 2. Although simplified Chinese characters were accepted for use many years ago, it seems that more and more people like Chinese characters in the complex form. 〖第八招〗注意熟词生义 英语中大量的词汇具有多义性,其含义并非完全等同于词典所标注的汉语意思,具体 的词义需要在一定的上下文中体现出来。阅读理解中的熟悉词生义比生词本身的障碍要大得 多。原因有三:其一,高考对生词的数量有量的规定,不得超过3%;其二,熟悉词生义很 容易引起考生的误解;其三,熟悉词生义的数量没有限制。如果这种现象在一篇文章中出现 得多,那就更难理解了。 下题中划线的词可换为: 1. New York, 10 November, 5:27 p.m.,yesterday. Biggest power failure in the city’s history. A. enough B. not passing C. lack of D. lack 2. A bike tour and race will be held in August 26 and 27. At 5:30 a .m. , the riders will leave Tian’an Men square and ride the first 35 kilometers as a training lge . Then the next 55 kilometers from Yanjiao to Jixian, will be the first competitive part for the tour. A. race B. practice C. part of the training D. part of the tour 〖第九招〗根据常识或经验猜测词义 在阅读中碰到生词时,我们有时可以运用逻辑推理能力、自身的生活经验及生活常识去 推断生词的含义。当然也要结合语境。 1. Rainforests like the Amazon are important for mopping up CO from the atmosphere and 2 helping to slow global warming. The underlined phrase “mopping up ” in the second paragraph means _______. A. cleaning up B. taking in C. wiping out D. giving up 2. She walked quietly to the small room by the lift and took a mop . She pushed past the desk and as the nurse looked up. Mum nodded and said, “Very dirty floor. 〖第十招〗利用词性转化 1. The most important reason for such a visit is to realize how our ancestors battled nature with the basic tools they had. 2. The aging of the population will affect American society in many ways-education, medicine, and business. The underlined work “aging” means ______. A. counting the number of years someone lives B. the numbers of years someone lives C. becoming older D. making someone looking older 【主旨大意】 这类题主要是测试学生对一篇文章或一段文字的深层理解程度及在速读中准确把握文章 主旨大意的能力。一般针对某一语段或某一语篇的主题标题或目的设题。常见的设题方式有: 1) Which of the following best describes the main point of the passage? 2) What is the writer trying to tell us? 3) Which of the following is the best title for the passage? 4) The purpose of the passage is. 5) The main (general) idea of the passage is. 6) The passage is mainly about. 7) What is mainly discussed in the passage? 这类题通常围绕一个中心思想展开,一些文章一开头便展示出文章的中心思想,第一段常常 是内容的梗概,同时又表达了中心思想,也有一些文章的中心思想贯穿全文,并没有用一句 话明确表达出来,这就要求学生学会归纳概括。每个段落往往也由一个主题句或几个陈述句 构成,它们在句中的位置不同,有时在开头, 首先点明本段大意; 有时在结尾, 总结本段大 意。 做这类题,首先要找出文章的主题句。找出文章的主题句,也就明确了文章要讲什么, 再通过速读全文,就可以把握文章的中心思想了。 主题句在整个语段中起着通领全段的作用,其它句子都是用来阐述、解释、支持或发展 主题句所表达的主题思想的,其位置一般位于段首,也见于段尾或段中。 另外在许多文段中,设有可以概括全段意义的主题句,必须根据文章中所提供的事实细 节,进行全面分析,然后归纳成一般概念。但必须注意,既不能以偏概全,也不能在概括时 过于宽泛,要恰如其分。 这就需要进一步加工概括了。 〖第一招〗 在许多情况下,尤其在阅读说明文和议论文时,根据其篇章特点我们可以通过寻找短文 的主题句来归纳出文章的主题。主题句在文章中的位置通常有三种情况:开头、中间、结尾 (含在开头结尾同时出现、首尾呼应的主题句)。因此仔细阅读这类文章或段落的首尾句是 关键。做主旨大意类试题多采用浏览法。浏览时,一般不需逐句浏览,只需选读文章的首段、 尾段,或每段的首句和尾句。重点搜索主题线索和主题信息。 有些文章的主题句或者说“文眼”出现在文章的最后,此类文章往往以列举事实开头,通 过论证,最后阐述核心观点,或者引用某个人的话印证自己的观点,以此归纳文章的主旨大 意,所以有时要找出这样的信息,从中提炼标题或归纳大意。 例如:河北唐山市高三第二次模拟考试题E篇: In recent years many countries of the world have been faced with the problem of how to make their workers more productive. Some experts claim the answer is to make jobs more varied. But do more varied jobs lead to greater productivity? There is evidence to suggest that while variety certainly makes the workers’ life more enjoyable, it doesn’t actually make him work harder. As far as increasing productivity is concerned, the variety is not an important factor. Other experts feel that giving the worker freedom to do his job in his own way is important and there is no doubt that this is true. The problem was that this kind of freedom cannot easily be given in fixed way. Thus freedom of choice may be important, there is usually very little that can be done to create it . Another important consideration is how much each worker contributed to the product he is making. In most factories the worker sees only one small part of the product. Some car factories are now experimenting with having many small production lines rather than one large one, so that each worker contributes more to the production of the cars on his line. It would seem that not only is the degree of workers’ contribution an important factor, therefore, but it is also one we do something about. To what extent more money led to greater productivity? The workers themselves certainly think this is important. But perhaps they want more money only because the work they do is so boring. Money just lets them enjoy their spare time more. A similar argument may explain demands for shorter working hours. Perhaps if we succeed in making their jobs more interesting, they will neither want more money, nor will shorter working hours be so important to them. 59. In this passage, the author mainly talks about ______. A. how to make the workers contribute more B. possible ways leading to greater productivity C. to what extent more money leads to greater productivity D. how to make workers’ jobs more interesting 〖第二招〗抓住文章段落大意,概括中心思想 寻找整篇文章的中心思想是建立在寻找具体段落中心的基础上的。各段落中心的整体归 纳便是文章的中心思想。在这个过程中,考生们不能只依据只言片语,或光看文章的某一些 段落,而应该观察全文的结构安排,理解文章的重点,考虑文章中材料及支撑性细节是服务 于什么的,分析故事的发展结局都是围绕什么中心来安排的。最后用简明扼要的文字将文章 的中心思想表达出来。即不能以偏概全,也不能在概括时过于宽泛,要恰如其分。 例如: Many people think a telephone is essential. But I think it’s a pest and a time waster. Very often you find it impossible to escape from some idle or curious chatterbox, or from somebody who wants something for nothing. If you have a telephone in your own house, you will admit that it tends to ring when you are asleep, of in the middle of a meal or a conversation, or when you are just going out, or when you are taking your bath. Are you strong-minded enough to ignore it, to say to yourself, “Ah, well, it will all be the same in a hundred year’s time?” You are not. You think there are maybe some important news or messages for you . I can assure you that if a message is really important it will reach you sooner or later. Haven’t you ever rushed dripping from the bath, or chewing from the table, or dazed from the bed, only to be told that you are a wrong number? But you will say, you need not have your name printed in the telephone directory, and you can have a telephone which is only unable for outgoing calls. Besides you will say, isn’t it important to have a telephone in case of emergency-illness, an accident, or fire? Of course you are right, but here in a thickly populated country like England, one is seldom far from a telephone in case of dreadful necessity. I think perhaps I had better try to justify myself by trying to prove that what I like is good. I admit that in different circumstances-If you were a business VIP, for instance, or a bed ridden person-I might find a telephone essential. But then if I were a taxi driver, I should find a car essential. Let me put it another way: there are two things for which the English seem to show particular talent. One is mechanical invention, the other is literature. My own business happens to be with the use of words but I see I must now stop using them. For I have just been handed a slip of paper to say that somebody is waiting to speak to me on the telephone. I think I had better answer it .After all, one never knows, it may be something important. 41. The passage is mainly about _______. A. that we should be strong enough to ignore a phone call B. that important message will reach you sooner or later C. whether it’s necessary to answer all telephone calls D. whether it is necessary to have a telephone 〖第三招〗抓住文章主线和关键词语,归纳文章中心 要注意不是所有的段落都有主题句,有时主题句暗含在句中。阅读这样的文章,就需 要自己根据文章的细节来分析,概括出段落的主题,从而推导出文章的主旨。分析的方法是: 先弄清该段落主要讲了哪几方面的内容,这些内容在逻辑上有什么联系,然后加以归纳形成 主题。如: Tom Brennan was working in a Philadelphia office building when he noticed a black bag. The bag contained a book. This chance discovery ended a 12-day search by the Library Company of Philadelphia for a historical treasurea 120-page diary dept 190 years age by Deborah Logan, “a woman who knew everybody in her day,” James Green, the librarian told the magazine American Libraries. Most of the diary is a record of big events in Philadelphia. It also includes a description of British soldiers burning Washington. D. C in the war of 1812. She describes President James Madison on horseback as “perfectly shaking with fear” during the troubled days. George Washington, she writes, mistook her for the wife of a French man and praised her excellent English. The adventure of the lost book began on September 4 when Cory Luxmoore arrived from England to deliver the diary of his ancestor to the Library Company, which he and his wife considered to be the best home for the diary. Green told American Libraries he had the diary in his possession “about five minutes” when Luxmoore took it back because he had promised to show it to one other person. On returning to his hotel after showing the precious book to Green, Luxmoore was shocked to realize that he had left it in the taxi. Without any delay, Green began calling every taxi company in the city, with no luck, “I’ve felt sick since then” Luxmoore told reporters. According to Green on one has yet learned how the diary came to the office building. Tom Brennan received a reward of 1,000, Philadelphia gained another treasure for its history, and Luxmoore told reporters, “It’s wonderful news. I’m on high.” 51. This article mainly tells about the story of “__________”. A. A lost diary B. Deborah C. Cory Luxmoore D. The Library Company 分析:解题时必须首先理顺文章时间顺序: 1. Deborah Logan kept a 120-page diary 190 years ago. 2. Cory Luxmoore delivered the diary to the library Company. 3. James Green, the librarian, had the diary in his possession about five minutes. 4. Cory Luxmoore left the diary in the taxi. 5. Tom Brennan noticed a black bag which contained a book (the diary). 【文章的标题】 标题位于文章之首,用来高度概括文章内容,点明文章主题。它是段落中心思想的最 精炼的表达形式,它的特点是:短小精炼,多为一个短语;涵盖性强,一般要求能覆盖全文, 其确定的范围要恰当,即不能太大,也不能太小,恰如帽子,太大能遮了眼睛,太小则发挥 不了应起的作用且风一吹必跑;精确性强,不能随意改变语言表达的程度及色彩。它可以是 单词,短语,也可以是句子。标题可帮助读者迅速推测出整篇文章的主要内容,抓住文章讨 论的中心,把握作者的观点和意图,那么如何选择文章的标题呢, 〖第一招〗 要在阅读原文的基础上,仔细考虑这句话或短语与文章主题是否有密切的联系;再看它对文 章的概括性或覆盖面如何;然后要注意题目是否过大或过小,要避免下列三种错误:?概括 不够(多表现为部分代整体,从而导致范围太小);?过度概括(多表现为人为扩大范围); ?以事实、细节替代抽象的大意。 要恰当地选好标题,还需要了解标题的基本拟定方式。一般来说,标题的拟定方式是:以话 题为核心,与控制性的概念词按一定的语法浓缩为概括句意或中心思想的词组。 例如:石家庄市高中毕业班第二次模拟考试题C篇 People say teenagers are no good. They make too much noise in shopping malls; they drive recklessly up and down America’s main streets; they carry chips on their shoulders as big as the Sears Tower. And at least some of the time those things are true. But we shouldn’t forget that there are hard moments in the life of a teenager too. I watched such a moment not long ago at a woman’s funeral. I didn’t expect the event to affect me. Through much of the ceremony, in fact I remained unmoved. Then her teenage grandson stepped forward. With his first very deep breath, every heart in that church was achingly reminded of something we had all forgotten. Softly he began: “I want to share a few values that Nana taught me. She never failed to see light in any situation. If our family dog should attack her, what would Nana say?” “Oh, what beautiful markings that dog has!” That was Nana. “She was a strong woman who often lived in the shadow of my grandpa, who was a successful businessman in the city. But she was the one behind the scenes who provided the strength and support for Grandpa’s career,” he said, with a voice now trembling. “That was Nana’s way.” Through a sob, he continued, “Whenever she did anything worth recognition, you’d have to hear about it from a different source, because she was never the one to brag.” Finally, in a voice breaking free of sorrow, he looked up and said, “Nana taught me courage. She put up an incredible fight to the end, when she died peacefully, which is how she lived her life. That was Nana’s way, and I hope I can carry on in the asme manner.” There are no hearts as sensitive as those of teenagers, because everything is happening to them for the first time. The trouble with the teenagers is that they haven’t learnt to be controlled. When that boy rose to speak about the woman who surely had truest ally and dearest friend, his honest voice dragged each of us out into the open where could no longer hide in the calm ritual. He exposed us to the truth about this very real woman who believed in a boy who probably tried the patience of many adults. He reminded us that his grandmother was more than another dot on the chart of life and death. All over again we felt those powerful losses crisscrossing our own hearts. As we know, when you say goodbye to a beloved grandparent, you say goodbye to something happy, something young in yourself, and even if something never really returns, the pain never really goes away. 52. Which of the following might be the best title for the passage? A. A Moving Funeral B. Are Teenagers Always Light-hearted? C. Teenagers Are No Good D. A Teenager And His Grandma 〖第二招〗 有些文章,比如记叙文,主题思想不是通过一两句话就可以表达的。这时就需要阅读 全文,抓住关键字眼或主要论题来归纳文章主题。 例如:河北省保定市高三模拟试题A篇 “Jingle bells, jingle bells, jingle all the way…” goes the popular Christmas tune. With the song in my head, I felt a bit sorry for myself. Instead of rushing back and forth across a freezing school campus, I could have had a cozy Christmas dinner at home or enjoyed the fabulous fireworks lighting up campus at night. But this Christmas would not be celebrated as usual. One month ago, all six girls in my dormitory (including me )had made detailed plans for a more-exciting-than-ever Christmas holiday. But one day we received an unusual mission: we were asked to organize a party that would be a fund-miser for mentally disabled children from the hospital. The children would perform on stage while volunteers offered food to the spectators and collect some money for the organization. Naturally, we were supposed to support the ideas wholeheartedly, and we did. But the date of the show was set right on Christmas Eve! This meant that all our original Christmas plans would have to be set aside. However, we took the plunge into preparations for the party. We spared no effort in organizing, designing, propagandizing, making contacts with the potential sponsors, negotiating and so on. We went through thick and thin and finally made it, though with a bit of reluctance. Finally, Christmas Eve arrived. As I dragged myself into the auditorium, I discovered the notes of a beautiful Christmas song that immediately filled me with happiness. I looked to find the source of the song… it was a group of children singing on stage. Though disabled, they appeared extraordinarily earnest and sincere in front of the audience. Everyone attending was deeply moved by the sight. People wanted to donate some money or show their support. The performance earned recognition and respect from all of us. At that point, I found that all I had done in the one-month time was worthwhile. And all my classmates who had been participating in this Christmas party shared the same feeling: getting the true meaning of Christmas and receiving the love from one another. 41. What would be the best title for the text? A. Christmas on stage B. An unusual Christmas C. Christmas party D. Volunteers for disabled children 【推理判断】 推理判断题要求考生在爱理解原文的基础上,根据文章所阐述的事实死结和上下问的 按时及线索进行综合分析,然后进行判断,引申或得出合情合理的结论。因此,考生不仅要 掌握文章的表层意思还要理解其深层的内涵,不仅要掌握已言明的观点和倾向,还要获得字 里行间的信息。 此题目难度大,涉及面广,如人物的性格、心理、故事的结局、寓意,文章的出处、体 裁,作者的倾向、态度等。考生因找准文中提供的相应的文字信息,特别是关键词语,把握 作者的观点、态度或写作的目的,借助上下文的逻辑关系来判断,并能领悟文字的弦外之音。 也可以借助常识或文化背景来判断。推理题在提问中常用的词有: infer, imply, suggest, indicate等。 推理判断常见的命题方式有: We can infer from the passage that the author _____. The main purpose of the survey is to ________. What is the author’s purpose in writing the passage? What would the author probably discuss in the passage that follows? The underlined sentenced in the second paragraph implied that ______ The passage is probably taken from a _______. 因此,常见的推测题有: 一、写作意图推测题: 此类题型要求考生根据文章的论述,推测作者的写作意图及应用某种写作手法的目的。作 者一般不直接陈述自己的意图,而是通过文章所提供的事实,客观地使读者信服某种想法或 意见。这类题型不仅要求考生能理解文章的内容,而且还要具备对作者所阐述的问题及写作 手法进行归纳总结和分析的能力。例如: Earthquake may rightly ranked as one of the most disastrous forces known to man: since record began to be written down, it has been estimated (估计) that earthquake-related deaths have numbered in the millions, and that earthquake-related loss has been beyond calculation… The great majority of all earthquake occurred in two particular areas in history… It is doubtful that man will be ever be able to control earthquakes and get rid of their damage altogether .But as how and why earthquakes happen become better understood, man will become more and more able to deal with their possible damage before they occur. Q: The author’s purpose in writing the passage probably is to ____. A. inform the reader B. entertain the reader C. support a theory D. question a theory 二、态度、观点判断题: 作者的态度、思想倾向是指对作者陈述的观点是赞同、反对还是犹豫不决,对记述和描 写的人、物或事件等是赞扬、同情、冷漠还是厌恶、憎恨。作者的这种思想倾向和感情色彩 往往隐含在文章的字里行间或流露于修饰的词语中。因此,在推理判断的过程中,因特别注 意作者的措辞。例如: Why isn’t your newspaper reporting any good news? All I read about is murder, bribery and death. Frankly, I’m sick of all the bad news. Q: The author’s attitude towards the newspaper reporting is to ______. A. complain B. apologize C. amuse D. inform 分析: 作者一是向读者说明这份报纸上只刊登不好的新闻,如凶杀、行贿、死亡等,二 是在字里行间流露出自己的抱怨情绪,故最佳答案是A 。 三、逻辑结论题: 此类题是严格根据文中所陈述的事实、论点进行推理,得出合乎逻辑的结论,而不是根 据自己的经验、态度、观点或爱好去理解文章的内涵。要首先获得短文的主题思想或列举的 事实,然后按要求判断。例如: Just as crying can be healthy, not crying — holding back tears of anger, pain or suffering — can be bad for physical (身体的) health. Studies have shown that too much control of emotions can lead to high blood pressure, heart problems and some other illnesses. If you have a health problem, doctors will certainly not ask you to cry. But when you feel like crying, don't fight it. It's a natural — and healthy — emotional response (反应). 61, It can be inferred from the text that ______. A(there are two ways to keep healthy B(crying does more good to health than laughing C(crying and laughing play the same roles D(emotional health has a close relationship to physical health 四、预测推断题: 有些题目要求考生根据语境对可能出现的结局或下段可能涉及的内容等进行预测推理。做 这类题目应把握作者的写作思路,从而作出比较科学的预测。例如: With the past in mind, preparations for 2006 season included sweeping changes on both the technical and personal sides. The management at Toyota Motorsport GmbH was restructed, with John Howett moving from Toyota Motor Marketing Europe to become President, thus enabling the company founder, Ove Anderson, more time to concentrate the operation of the race team at the track. Anderson’s value as an inspirational leader was one of the team’s biggest asserts(财富) in 2005. Q: What will the next paragraph talk about if the passage goes on? A. Preparations for the 2006 season. B. The value of Ove Anderson C. How to improve the European market? D. Changes on technical side 五、细节推断题 要求学生根据语篇关系,推断具体细节,如时间、地点、人物关系、人物身份、事件、具体 信息等。判断推理是一种创造性的思维活动。但它并非无章可循。 推理判断题要在阅读理解整体语篇的基础上,掌握文章的真正内涵。?要吃透文章 的字面意思,从字里行间捕捉有用的提示和线索,这是推理的前提和基础;?要对文字的表 面信息进行挖掘加工,由表入里,由浅入深,从具体到抽象,从特殊到一般,通过分析、综 合、判断等,进行深层处理,符合逻辑地推理。?要忠实于原文,以文章提供的事实和线索 为依据。立足已知,推断未知。 例: (2011?江苏卷B篇) Your National Park watchdog Attractions Do you: ?Love the National Park,value it and hope to safeguard its future? ?Wish to see the beautiful landscape of the Park protected? ?Like to enjoy peaceful,informal recreation within the Park? Aims The Friends organization aims are to help protect and improve the Pembrokeshire Coast National Park for all to enjoy.We are a voluntary organization and registered charity without financial links to the National Park Authority. Activities We encourage everyone to enjoy the National Park through regular talks and visits to interesting places in the Park with expert guides. We keep an eye on planning applications,Park Authority policies and threats to the National Park such as massive leisure complexes.We work with like,minded organizations such as the Campaign for National Parks to make our voice more effective. We help children to understand the National Park by sponsoring publications such as an adventure booklet and projects in local schools. Benefits ?Guided visits to places of interest which may not always be available to the general public. ?All members receive our regular News and Views. ?Talks by experts in their fields on current issues. ?A discount is available on Friends items for sale. ?Satisfaction of participation in work parties, for those willing and able to be involved. If interested,please complete the Application Form at www.fpnp.org.uk. 60. Which of the following is discouraged by the Friends organization? A. To build massive complexes for public amusement. B. To prevent possible damages to the National Park. C. To help protect and improve the Park for all to enjoy. D. To sponsor publications and projects in local schools. 考生做题时一定要整体理解语篇内容,在语篇的表面意义与隐含意义、已知信息与未 知信息间架起桥梁,透过字里行间,去把握作者的“弦外之音”。 首先,考生在阅读时,要抓住文章的主题和细节,分析文章结构,根据上下文内在联 系,挖掘文章的深层含义。在进行推理时,一定要仔细阅读短文,千万不可脱离原文,仅凭 个人的看法主观臆断。 其次,对于暗含在文章中的人物的行为动机、事件中的因果关系及作者未言明的倾向、 意图、态度、观点等要进行合乎逻辑的判断、推理、分析,进一步增强理解能力,抓住材料 实质性的东西。 第三,在解答推理性问题时,应清楚所要解答的问题需要针对某个细节进行推断,还 是针对主题思想、作者的意图进行推断。针对细节的推断可运用canning的方法,迅速在材 料中确定推理依据的位置或范围,然后再进行推理判断。针对主题思想作推断时,其解题的 主要依据是文章的主题思想,然后再分析句子之间的逻辑关系,区分观点与例证(opinion and fact)、原因与结果(cause and effect)、主观点与次观点(main idea and supporting idea)。 〖第一招〗抓住特定信息进行逆向或正向推理 做此类试题要善于抓住某一段话中的关键信息,即某些关键词或短语去分析、推理、 判断,利用逆向思维或正向推理,推断出这句话所隐含的深层含义。 〖第二招〗 整合全文(段)信息进行推断 做推理题时,有时需要在弄懂全文意思的基础上,整合与题目有关的有用信息,综合 起来去推理判断,确定最佳结论,忌“见树不见林”。 例如: ?Brain-injured boy set for 1.75m payout A YOUNGSTER who was knocked off his tricycle by a serious brain injury is set to be awarded 1.75 million. Fourteen-year-old George Currie, from Dalkeith was living in County Durham with his parents Allister and Paula at the time of the accident in September 1993. George, who was then three years old, was riding his small tricycle along the pavement(人 行道)when he lost control and swerved (突然转向) on to the road into the path of an on-coming taxi. The driver’s insurance(保险) company has now accepted responsibility for the accident and New Castle County Court is expected to rule on the payout on Monday. ? TOP 10 Florida counties with the most accidents in 2004: 1. Monroe (98) 2. Palm Beach (65) 3.Pinellas (64) 4. Broward (59) 5. Miami-Dade(54) 6.Okaloosa (34) 7. Orange (24) 8. Bay (22) 9. Lee (21) 10. Collier (21) Boating accidents reduce Lee and Collier counties tied at No. 10 for boating accidents in Florida in 2004, an improvement for both. But because the counties remained in the top 10 among Florida’s 67 counties, safety continues to be a concern, officials said. ? Teens react to new law on driving permit Lawmakers may hope to make teenage drivers drive more safely, but tome local teens behind the wheel feel angry following the passing of Senate Bill 36 last week. SB 36 is a law that limits times new teen drivers can drive and the number of passengers they can carry. ? Road accidents take costly toll Traffic accidents kill more than one million people each year, injure tens of millions more and cost developing countries twice as much as they receive in international aid, the leader of a research body said Wednesday. The figures have led experts to form an international roadsafety network to choose the most important research areas, inform policy makers and help develop ways to prevent accidents, particularly in poor nations. 75. Which passage would give more intormation on the prevention of road accidents? A. ? B. ? C. ? D. ? 〖第三招〗 利用语境的褒贬性进行信息推断 几乎每篇文章的语境都有一定的褒贬性,这种褒贬性反映了主人公的特定心理和情绪 状态及作者的写作意图,因此利用好文章的语境褒贬性及变化的标志性词汇或句子对我们掌 握文章主旨大意,了解主人公的心理特点及发展变化,从而正确地进行逻辑推断很有好处。 例如: Is it possible to persuade mankind to live without war? War has existed for at least six thousand years. It was always evil and usually foolish, but in the past the human race managed to live with it. Modern skill has changed this. Either man will end war, or war will end man. For the present, it is nuclear weapons(核武器) that cause the greatest danger, but bacteriological or chemical weapons may, before long, offer an even greater threat. If we succeeded in stopping war, there would be no danger for us. To do this, we need to persuade mankind to look upon international questions in a new way, not by contests of force, in which the victory goes to the side which is most skillful in killing people, but by talking in accordance with agreed rules of law. It is not easy to change old mental habits, but this is what must be attempted. Some people hold completely false attitudes towards how to prevent war. In fact, they are willing to go to war in support of them. However the movements of the world opinion during the past two years has changed very largely such as we can welcome. It has become a common belief that nuclear war must be avoided. Of course, very difficulit problems remain in the international world but the attitude towards them is a better one than it was some years ago. It has begun to be thought, even by the powerful man who decide whether we shall leave or die, that agreements should be reached, even if both sides do not find these agreements wholly satisfactory. It has begun to be west, but between man and the bomb. 56. from the first paragraph we can know that _____. A. we may face greater threat from weapons B. chemical weapons are less dangerous than nuclear weapons C. man’s idea of victory has changed D. dangerous weapons are forbidden in modern society 58.The author believes that the only way to stop war is to ____ A. stop nuclear weapons B. settle international problems through agreements C. get rid of bacteriological and chemical weapons D. let the stong side take over the world 60. which of the following words can best describe the author’s feeling in writing this passage? A. disappointed B. doubtrul C. worried D. hopeful 〖第四招〗 根据文章的结论推断作者的态度 作者的态度、倾向是指作者对陈述的观点是赞同、反对还是犹豫不定,对记叙或描写 的人、物或事件等是赞颂、同情、冷漠还是厌恶、憎恨。作者的这种思想倾向和感情色彩往 往隐含在文章的字里行间,或流露于修饰的词语之中。因此,在推断过程中,应特别注意文 中作者的措辞,尤其是表达感情色彩的形容词。 做这一类题时一定注意: (1)由表及里地准确把握字里行间的意思,不能主观臆断,凭空想象,随意揣测,更不 能以自己的观点代替作者的观点。 (2)特别注意那些描写环境气氛的语言,以及表达感情、态度观点的词语。要文章 的基调,揣摩作者的态度,悟出作者的弦外之音。 (3)能结合自己平时积累的有关英语国家的文化传统、风俗习惯等背景知识来识别评价。 人物性格、态度及观点判断题 例:(2011?江苏卷A篇)We know the famous ones — the Thomas Edisons and the Alexander Graham Bells — but what about the less famous inventors? What about the people who invented the traffic light and the windshield wiper(雨刮器)? Shouldn't we know who they are? Joan McLean thinks so. In fact, McLean, a professor of physics at Mountain University in Range, feels so strongly about this matter that she's developed a course on the topic. In addition to learning “who” invented “what”, however, McLean also likes her students to learn the answers to the “why” and “how” questions. According to McLean, “When students learn the answers to these questions, they are better prepared to recognize opportunities for inventing and more motivated to give inventing a try.” Her students agree. One young man with a patent for an unbreakable umbrella is walking proof of McLean's statement. “If I had not heard the story of the windshield wiper's invention,” said Tommy Lee, a senior physics major, “I never would have dreamed of turning my bad experience during a rainstorm into something so constructive.” Lee is currently negotiating to sell his patent to an umbrella producer. So, just what is the story behind the windshield wiper? Well, Mary Anderson came up with the idea in 1902 after a visit to New York City. The day was cold and stormy, but Anderson still wanted to see the sights, so she jumped aboard a streetcar. Noticing that the driver was struggling to see through the snow covering the windshield, she found herself wondering why there couldn't be a built,in device for cleaning the window. Still wondering about this when she returned home to Birmingham, Alabama, Anderson started drafting out solutions. One of her ideas, a lever(操作 杆)on the inside of a vehicle that would control an arm on the outside, became the first windshield wiper. Today we benefit from countless inventions and innovations. It's hard to imagine driving without Garrett A. Morgan's traffic light. It's equally impossible to picture a world without Katherine J. Blodgett's innovation that makes glass invisible. Can you picture life without clear windows and eyeglasses? 56. By mentioning “traffic light”and “windshield wiper”, the author indicates that countless inventions are ______. A. beneficial, because their inventors are famous B. beneficial, though their inventors are less famous C. not useful, because their inventors are less famous D. not useful, though their inventors are famous 〖第五招〗根据上下文的逻辑得出结论 逻辑结论是指严格根据文章中所陈述的事实、论点、例证等一系列论据材料进行推理, 从而得出合乎逻辑的结论,而不是根据自己的经验、态度、观点或爱好去理解文章的内涵。 做这类题时应把握作者的写作思路,预测下文可能发展的内容。文章可能按事件发展的经过 描写,也可能按因果关系,对比关系来描写。必须注意的是:1、读者不可根据自己的主观 臆断毫无根据地下结论;2、所得结论必须符合作者的意图。 例如:NMET2000E篇 66. According to the writer of the text, imagining the future will ____. A. serve the interests of the present and future generations B. enable us to better understand human’s history C. help us to improve farming D. make life worth living 分析:首先作者提出自己的观点:However, most of our ideas about the future are really very short-sighted… so why bother even to try imagining life far in the future?然后提出正反两个 次要观点去论述:First unless we remember how shout our own lives are compared with the whole human history, we are likely to think our own interests are much more important than they really are. Second, by trying to escape from present interests and imagining life far in the future we may arrive at quite fresh ideas that we can use ourselves(take action right away)(to make the world a better place in which to live).由第二个次要观点可判断上题答案为A。 〖第六招〗 结合已有的知识进行推断 知识推断是根据文章中所阐述的细节,运用自己掌握的基础知识进行分析、推敲,从 而得出符合文章原意的结论的一种推断方法。知识推断题一般都针对细节。解答此类题。不 仅需要我们有一定的社会基础知识,而且需要对文章细节以及有关背景知识有一个充分的理 解。 例如:NMET1996A篇 53. After reading the story what can we infer about the hospital? A. It is a children’s hospital B. It has strict rules about visiting hours C. The conditions there aren’t very good D. The nurses and doctors there don’t work hard 分析:本文讲述了一位母亲采用拖地计谋混进病房近视女儿的故事。由常识可知医院 内拖地并非医生护士的工作,选项D不对。C项与文中的it’s a fine hospital意思相矛盾。A 项依据不充分,女儿未必是儿童。正确答案是B。文中I told her about the hospital rules, and she will not expect us until tomorrow则暗示此规章在探视时间上是严格规定的,而且在实际 生活中我们都知道医院对探视病人是有严格规定的。 〖第七招〗 根据文章的篇章结构去进行判断推理。 有些内容文章中没有明确说明,要求考生根据语篇,对事件可能的结局或下段可能涉及 的内容等进行预测推理。做这类题时应把握作者的写作思路(如文章可能按事件发展的经过 描写,也可能按因果关系、对比关系来叙述),要把握句、段之间的逻辑关系,了解语篇的 结构。摸准逻辑发展的方向,从而作出比较科学的、合情合理的预测。 文章思路、结构、写作意图及 写作方法等推测提 有时题目要求考生根据文章的论述,推测作者的写作意图、短文组织结构及运用某种写作手 法的目的。作者一般不直接陈述自己的意图,而是通过文章所提供的事实和形象,客观地使 读者信服某种想法或意见。这种题型要求同学们不但能理解文章的内容,同时还要具备对作 者阐述问题的写作方法及文章结构进行归纳总结和分析的能力。 例:(2011?四川卷D篇)“Experience may possibly be the best teacher, but it is not a particularly good teacher. ” You might think that Winston Churchill or perhaps Mark Twain spoke those words, but they actually come from James March, a professor at Stanford University and a pioneer in the field of organizational decision making. For years March (possibly the wisest philosopher of management) has studied how humans think and act, and he continues to do so in his new book The Ambiguities of Experience. He begins by reminding us of just how firmly we have been sticking to the idea of experiential learning: “Experience is respected; experience is sought; experience is explained.” The problem is that learning from experience involves(涉及) serious complications(复杂化),ones that are part of the nature of experience itself and which March discusses in the body of this book. In one interesting part of the book,for example,he turns a doubtful eye toward the use of stories as the most effective way of experiential learning. In our efforts to make stories interesting, he argues, we lose part of the complicated truth of things. He says “The more accurately(精确 的)reality is presented,the less understandable the story,and the more understandable the story, the less realistic it is. ” Besides being a broadly knowledgeable researcher, March is also a poet, and his gift shines through in the depth of views he offers and the simple language he uses. Though the book is short, it is demanding: Don't pick it up looking for quick, easy lessons. Rather, be ready to think deeply about learning from experience in work and life. 55(What can we learn from Paragraph 3? A. Experience makes stories more accurate. B(Stories made interesting fail to fully present the truth. C(The use of stories is the best way of experiential learning. D(Stories are easier to understand when reality is more accurately described. 56(What's the purpose of this text? A(To introduce a book. B(To describe a researcher. C(To explain experiential learning. D(To discuss organizational decision making. 有时,阅读理解题还要求考生推断所读材料的来源或所读材料的读者对象。判断材料的来源 主要根据文章的体裁和题材。一般来说,广告类,时事新闻类,娱乐新闻类,影评,应用文 类通常出现在报纸、杂志、海报上。说明文通常出现在教科书或实用技术手册里面。判断读 者对象主要通过寻找关键的信息词。 来源、读者对象推测题 【示例1】 Edward Wilson is America’s, if not the world’s, leading naturalist. In The Future of Life, he takes us on a tour of the world’s natural resources (资源). How are they used? What has been lost? What remains and is it able to continue with the present speed of use? Wilson also points out the need to understand fully the biodiversity (生物多样性) of our earth. Wilson begins with an open letter to the pioneer in environment (环境) protection Henry David Thoreau. He compares today’s Walden Pond with that of Thoreau’s day. Wilson will use such comparisons for the rest of the book. The problem is clean: man has done great damage to his home over the years. Can the earth, with human help, be made to return to biodiversity levels that will be able to support us in the future? Biodiversity, Wilson argues, is the key to settling many problems the earth faces today. Even our agricultural crops can gain advantages from it. A mere hundred species (物种) are the basis of our food supply, of which but twenty carry the load. Wilson suggests changing this situation by looking into ten thousand species that could be made use of, which will be a way to reduce the clearing of the natural homes of plants and animals to enlarge farming areas. At the end of the book, Wilson discusses the importance of human values in considering the environment. If you are to continue to live on the earth, you may well read and act on the ideas. 72. We learn from the text that Wilson cares most about . A. the environment for plants B. the biodiversity of our earth C. the waste of natural resources D. the importance of human values 73.How many species are most important to our present food supply? A. Twenty. B. Eighty C. One hundred. D. Ten thousand. 74.Wilson suggests that one way to keep biodiversity is to . A. learn how to farm scientifically B. build homes for some dying species C. make it clear what to eat D. use more species for food 75.We can infer that the text is . A. a description of natural resources B. a research report C. a book review D. an introduction to a scientist 【练习1】 WANG Hao and Wang Liqin, two of China's top table tennis players, both saw the World Table Tennis Championships as a chance to prove themselves. But there was only one champion (冠军) and Wang Hao, 25, won it last Tuesday in Yokohama , Japan. Wang Hao, who won the men's doubles title with Chen Qi last Monday, played with spirit to beat Wang Liqin 11-9, 13-11, 11-5, 11-9. Wang Hao said it was his mental toughness (心理稳定) that made the difference. "I was disappointed by my performances in past championships and the last two Olympic Games. But I have tried to pull myself together and prepared well. This is the result." Wang Hao said he has always understood the importance of physical training and technical skills. Now he knows the need for mental preparation as well. Having two Olympic silvers had left him feeling bitter (痛苦的). The bitterness perhaps explains why he has not always played at his best in the past. "I've come to understand that you can learn important lessons from failure," he said. "Losing can build your character and make you a stronger player." His next goal, or dream, is to win the gold medal at the London Olympics in 2012. Liu Guoliang, head coach of China's table tennis team, sang the praises of Wang Hao's victory. "This is just the beginning of his rise," said Liu. "It's the confidence-booster he needs to make further progress." 1. According to the passage, Wang Hao won _____ gold medals in the World Table Tennis Championships in Yokohama, Japan. A. only one B. two C. three D. four 2. From Wang Hao, we can know the main reason for his beating Wang Liqin is ______. A. his technical skills B. his physical training C. his mental toughness D. his past experience 3. What is the next goal of Wang Hao? A. To win the title of the champion at the 2012 Olympics B. To beat Wang Liqin again C. To prove himself in the future competitions D. To win more golds in future 4. What can we learn from Wang Hao? A. Failure is the mother of success. B. Never give up until you succeed. C. Where there is a will, there is a way. D. A good beginning makes a good ending. 【示例2】 Dear Hamilton, We are fortunate that in such a large, high-pressure office we all get along so well. You are one of the people who keep the social temperature at such a comfortable setting. I don’t know anyone in the office who is better liked than you. You can perhaps help with this. The collection of contributions towards gifts for employees’ personal-life events is becoming a little troubling. Certainly, the group sending of a gift is reasonable now and then. In the past month, however, there have been collections for two baby shower gifts, one wedding shower gift, two wedding gifts, one funeral(葬礼)remembrance, four birthday gifts, and three graduation gifts. It’s not only the collected-from who are growing uncomfortable (and poor), but the collected-for feel uneasy receiving gifts from people who don’t know them outside the office, who wouldn’t even recognize their graduating children, their marrying daughters and sons, or their dead relatives. This is basically a kind gesture (and one that people think well of you for), but the practice seems to have become too wide-ranging and feels improper in today’s office setting. Thank you for understanding. 63. The underlined word “contributions” probably means ________. A. money B. suggestions C. reports D. understanding 64. Hamilton is expected to _______. A. show more kindness. B. discontinue the present practice C. quit being the organizer for gift giving D. know more about co-workers’ families 65. This is basically a letter of ________. A. apology B. sympathy C. appreciation D. dissatisfaction 【练习2】 Have you ever used solar energy to boil a bottle of water or take a shower? Have you ever thought that solar energy is far away from our daily lives? You may doubt about that. In Japan, there is a wonderful program of Solar City. Solar City is located 80,000 meters northwest of Tokyo, among the strawberry fields. This is one of Japan’s sunniest places. Solar City covers about 410,000 square metres. Seventy-five percent of the homes there are covered by solar panels. The government has provided people there with the solar panels for free. In addition, the government has paid 9.7 billion yen to study how to make full use of solar energy. A number of solar energy companies have also given money to help this program. People living in Solar City think using solar panels is a good way to save money. However, saving money is not the only reason why people are moving into this city. "We moved here because of the panels. It was something we wanted,” said Mika Hiroshima. She moved to Solar City with her husband and two little children about four years ago. It is well known that Japan is a country that is short of energy, but the Solar City program has brought hope for the country. "People want solar energy,” said an official of Solar City. Actually in Solar City, there are 550 families making use of solar energy at the moment. All of them say that they want to keep using solar energy even after the program ends in 2010. In sunny days, solar panels are able to provide more than enough energy for a normal family. However, the solar panels are not that useful in cloudy days. Despite this, Solar City is still a wonderful program because it inspires people to wisely use the limited energy. 1. The writer uses the two questions at the beginning of the passage to ______. A. test the readers’ knowledge about solar energy B. draw the readers’ attention to the topic C. learn the situation that solar energy is used D. invite the readers to answer them 2. The third paragraph of the passage is mainly about ______. A. the reason why people are moving into Solar City B. how the people are living in Solar City C. the things that people living in Solar City need D. the life experience from the people living in Solar City 3. It can be inferred from the passage that _________. A. Japan is a country which is lacking in energy B. the solar panels are only useful in sunny days C. the Solar City program will be successful D. Solar City is a very modern big city 【示例3】 People believes that climbing can do good to health. Where can you learn the skill of climbing then? If you think that you have to go to the mountains to learn how to climb, you’re wrong. Many Americans are learning to climb in city gyms(体育馆). Here, people are learning on climbing. The climbing wall goes straight up and small holding places for hands and feet. How do people climb the wall? To climb, you need special shoes and (保护带) around your chest to hold you. There are ropes(绳索)tied to your. The ropes hold you in place so that you don’t fall. A beginner’s wall is usually about 15 feet high, and you climb straight up. There are small pieces of metal that stick out for you to stand on and hold on to. Sometimes it’s easy to see the new piece of metal. Sometimes, it’s not. The most difficult is an your fear. It’s normal for humans to be afraid of falling, so it’s difficult not to feel fear. But when you move away from the wall, the and the ropes hold you, and you begin to feel safe. You move slowly until you reach the top. Climbing attracts people because it’s good exercise for almost everyone. You use your whole body, especially your arms and legs. This sport gives your body a complete workout. When you climb, both your mind and your body can become stronger. 64. What can we infer from the passage? A. People are fairly interested in climbing nowadays. B. It is impossible to build up one’s body by climbing. C. People can only learn the skill of climbing outdoors. D. It is always easy to see holding places in climbing. 65. The most difficult thing to do in wall climbing is _______. A. to tie ropes to your B. to control your fear C. to move away from the wall D. to climb straight up 66. The word “workout” underlined in the last paragraph most probably means _________. A. settlement B. exercise C. excitement D. tiredness 67. Why does the author write this passage? A. To tell people where to find gyms. B. To prove the basic need for climbing C. To encourage people to climb mountains. D. introduce the sport of wall climbing 【练习3】 The only survivor of a shipwreck was washed up on a small, uninhabited island. He prayed feverishly for God to rescue him, and every day he scanned the horizon for help, but none seemed forthcoming. Exhausted, he eventually managed to build a little hut out of driftwood to protect him from the elements, and to store his few possessions. But then one day, after looking for food, he arrived home to find his little hut in flames, the smoke rolling up to the sky. The worst had happened; everything was lost. He was stunned with grief and anger. "God how could you do this to me!" he cried. Early the next day, however, he was awakened by the sound of a ship that was approaching the island. It had come to rescue him. "How did you know I was here?" asked the weary man of his rescuers. "We saw your smoke signal," they replied. It is easy to get discouraged when things are going bad. But we shouldn't lose heart, because God is at work in our lives, even in the midst of pain and suffering. Remember, next time your little hut is burning to the ground it just may be a smoke signal that summons the grace of God. For all the negative things we have to say to ourselves, God has a positive answer for it . 1(The proper title of the passage is . A(God Is Wherever We Need Him B(We should have a Positive Attitude to Life C(A Story Happened on An Uninhabited Island D(A Survivor Was How to Be Saved 2(After the survivor seeing his hut burning to the ground, he _________. A(was very angry and frightened and didn’t sleep B(was very sad and disappointed and didn’t sleep C(was so angry and sad that he slept without consciousness D(was so surprised that he slept without consciousness 3(From the story we can infer _______. A(that the survivor was a lucky dog B(God is a kind and considerable old man C(God is willing to help anyone who helps oneself D(You are what you eat 4. The meaning of the underlined part“ a smoke signal that summons the grace of God” may be___. A(a smoke signal that shows God’s idea B(a piece of information that God sends out to inform rescuers. C(an order that God gives to punish the man who is in trouble D(a signal that shows God’s kindness to help the man in trouble 实战练习(限时:每篇7分钟) 人物传记型阅读理解 (一) Elizabeth Blackwell was born on February 3, 1821 in Bristol, England. Her father was a rich sugar businessman at the time. Because her parents thought boys and girls should be equal, Elizabeth received the same education as her brothers. In 1832, her father?s business was destroyed by fire, so her family moved to New York City. But her father?s business there failed. Then in 1837, the family moved to Cincinnati, Ohio. Not long after, her father died. After her father?s death, Elizabeth, at the age of 16, had to go to work. When she was 24, she visited her dying friend Mary. Her friend said, “You?re young and strong, you should become a doctor.” That was nearly impossible for a woman in the middle of the nineteenth century. But she knew this was what she was going to do. After several rejections from medical schools, she finally was accepted by Geneva Medical College. By studying hard, she graduated successfully in 1849. After graduating from medical school, she went to Paris to learn more about medicine. She wanted to be a surgeon, but a serious eye problem forced her to give up the idea. When she returned to America in 1851, she found it difficult to start her own practice because she was a woman. In 1857, Elizabeth and her sister, also a doctor, along with another woman doctor, managed to open a new hospital, the first for women and children. Besides, she also set up the first medical school for women in 1868, where she taught the women students about disease prevention. It was the first time that the idea of preventing disease was taught in a medical school. Elizabeth Blackwell started the British National Health Society in 1871, which helped people learn how to stay healthy. In 1889, Elizabeth Blackwell became the first woman doctor in the United States. Most importantly she fought for the admission of women to medical colleges. Elizabeth Blackwell died on May 3, 1910, when she was 89.She opened a world of chances for women. She always fought for what was right in all her life. In 1949 the Blackwell medal was established. It?s given to women who have excellent achievements in the field of medicine. She?ll always be remembered as a great woman. ( )6.According to the passage, Elizabeth Blackwell ________. A(received bad education in her childhood B(spent a happy and lucky childhood C(moved to America with her family at eleven D(decided to be a doctor due to her father?s death ( )7.Elizabeth Blackwell could not become a surgeon because ________. A(she was a woman B(she had a serious eye problem C(she went to Paris for further education D(she didn?t go to medical school ( )8.Elizabeth Blackwell opened her first hospital for woman and children ________. A(eighteen years after her father died B(seven years after returning to America from Paris C(ten years before setting up her first medical school D(eight years after graduating from medical school ( ) 9.Elizabeth Blackwell spent most of her life in ________. A(the U(S. B(Paris C(England D(Geneva ( )10.Which of the following is not TRUE about Elizabeth Blackwell? A(She built the first hospital for women and children with others. B(She became the first woman doctor in the U(S. C(She set up the first medical school for women in the world. D(She built a medal for women with excellent achievements in medicine. (二) Ted Turner achieved high goals and great success by the time he was 43.Best known for his CNN, Ted Turner launched a second nationwide all-news network, Headline News, purchased the Atlanta Braves baseball team, and became the world?s best yachtsman at the America?s Cup in 1977.In 1982 Ted Turner was named by Forbes one of the 400 richest people in the U(S.He was named Time magazine?s “Man of the Year” in1992.How did Ted Turner accomplish all of this at such a young age? Ted Turner?s father, Ed, was a self?made millionaire who demanded that his only son try to achieve similar success.He instilled in the boy a strong belief that hard work was good.Besides reading a new book every two days, Ted Turner was also charged rent at home during summer vacations from boarding school.Ed Turner was a strong influence in his son?s life. As a child, Ted Turner lived a very lonely life, often separated from his family. During World War ?, his father served in the Navy. Ed Turner took his wife and daughter with him so they could live nearby but left his 6-year-old son behind in a boarding school in Cincinnati, Ohio. When Ted Turner was in the fifth grade, his father enrolled him in a military academy. Even though Turner had friends at school, it didn?t make up for the absence of his parents and sister. Eventually Ted Turner enrolled at Brown University in Providence, Rhode Island. He attended college off and on, became involved in sailboat racing, and became a member of the U(S. Coast Guard for a while. In his early twenties, he became general manager of one of his father?s branch offices—the Turner Advertising Company in Macon, Georgia. Two years later after his father?s suicide, Ted Turner took over the company.He soon discovered that he was more skilled than his father in managing the business.During the next twenty years, Ted Turner worked hard to accumulate enough power and money to fulfill his father?s dream. Reflecting on his father?s death, however, Ted Turner realized that it was dangerous to put too much emphasis on material possessions.He decided to use his hard-earned influence to serve the public. Concerned about the environment, Ted Turner established the Better World Society in 1985.The purpose of this organization was to produce documentaries to educate people about pollution, hunger, and the danger of building weapons of mass destruction.In 1986 Turner began sponsoring the Goodwill Games to promote world peace.The Turner Tomorrow Awards were created to encourage writers and thinkers to focus their attention on solving world problems.The Turner Family Foundation was established in 1992. ( )11.What is CNN according to the passage? A.It?s a broadcast?news company. B(It?s a basketball team agency. C.It?s a charity organization. D(It?s a publishing house. ( )12.What is the main idea of Paragraph 1? A.The creation of CNN encouraged broadcasters to be more successful. B.Towering goals and hard work led to fame and prosperity. C.Ted Turner was expert at sailboat racing and playing basketball. D.Ted Turner became very successful at an early age. ( )13.As a student in a military academy, Turner ________. A.wasn?t as lonely as he looked because of his many friends at school B.missed his family very much despite having friends at school C.served as a member of the U(S. Coast Guard for a while D.showed little interest in studying but other affairs ( )14.The author seems to believe that Ted Turner is ________. A.bossy B(incapable C(ambitious D(modest ( )15.Which of the following best describes an important change in Ted Turner?s life? A.He became more determined and was feared by his employees. B.He developed respect and admiration for his employees. C.He revised his view of the world and his responsibility in it. D.He was demanded the same level of success as his father. 故事记叙型阅读理解 (一) Our Community One Tuesday evening in the beginning of the fall 1996 semester (学期) at Shippensburg University, sirens sounded. These sirens were not in celebration; they were a cry to the university that something was wrong. A house, only one block away, was on fire. Nine of the university?s students lived there. From the minute the word got out that help was needed, it seemed like everyone showed up. The victims (受灾者) of the fire were offered endless invitations for housing for the night. The very next day, everyone got_into_gear to do their part in helping them. Flyers(, handbills) were posted with items that were immediately needed, just to get these students through this next couple of days. Boxes for donations (捐赠品) and money jars were placed in every residence hall (学生 宿舍)( As a residence director, I went before the students in my hall to ask them to do what they could. I knew that college students don?t have much, but I asked them to do their best: “Every little bit will help.” I really didn?t think they could do much. I was proved wrong. At the hall council meeting the night after the fire, my residents decided to have a wing competition, where each wing of the building would team up to see who could bring in the most donations. I announced that the wing who won would receive a free pizza party. Thursday evening we announced over the PA system that we were beginning the wing competition. Within minutes, the place exploded. The single large box that I had placed in the lobby (大厅) was overflowing. We quickly grabbed more boxes, and we watched in_amazement as they, too, filled to the brim(边;沿). Members of the resident assistant staff and I began to count the items. I was astonished by what I saw, and I was inspired by these kids. When we came to the final tally(得分), the winners turned to me and announced that they would like to donate their winnings as well. They wanted the victims of the fire to have their pizza party. Tears welled up in my eyes. I had watched these students jump to action, work tirelessly and donate all that they could. And then, as if that were not enough, they handed over their reward. I was touched and so very proud of them. ( )1.The writer mainly wanted to________ by the story. A(tell us nine of the college students suffered from a fire B(sing praise of the college students helping the victims selflessly C(describe how successful the wing competition was D(express he was deeply moved by the college students? action ( )2.The phrase “get into gear” in the second paragraph means ________. A(start working B(take measures C(be ready D(be eager ( )3.At first, the residence director thought ________. A(it was impossible to expect the college students to donate anything B(it was easy to collect a lot of donations from the college students C(the college students would donate something, but not much D(the college students would donate all they could ( )4.The wing competition was held ________. A(several days after the fire B(the night after the fire C(a week after the fire D(two days after the fire ( )5.The phrase “in amazement” in Paragraph 5 means ________. A(satisfactorily B(excitedly C(surprisedly D(happily (二) Martin was returning to work in his London office after spending two weeks with his brother in New York. He was coming back with a heavy heart. It was not just that it was the end of a wonderful holiday; it was not just that he invariably suffered badly from jet lag; it was that Monday mornings always began with a team meeting and, over the months, he had grown to hate them. Martin was aware that colleagues approached these meetings with hidden agendas; they indulged(纵容) game-playing; and he knew that people were not being honest and open. The meetings themselves were bad enough—there was all the moaning afterwards at the meeting like “I could have improved on that idea, but I wasn?t going to say”. As this morning?s meeting began, Martin prepared himself for the usual dullness and boredom. But, as the meeting progressed, he became aware of a strange background noise. At first, he thought that he was still hearing the engine noise from the aircraft that had brought him back to London. But, as he concentrated on the noise, it became a little clearer. He realized—to his amazement—that he could actually hear what they were thinking at the same time as they were speaking. What surprised him, even more than the acquisition(获得) of this strange power, was that he discovered that what people were saying was not really what they were thinking. They were not making clear their reservations. They were not supporting views which they thought might be popular. They were not contributing their new insights. They were not volunteering their new ideas. Martin found it impossible not to respond to his new knowledge. So he started to make gentle interventions, based more on what he could hear his colleagues thinking than on what he could hear them saying. “So, John are you really saying…” “Susan, do you really think that,” “Tom, have you got an idea on how we could take this forward,” They looked at him, puzzled. In truth, he felt rather proud of his_newly-acquired_talent. As the meeting progressed, it was clear to him that each member of the meeting was learning how to hear the thoughts of the others. The game-playing started to fall away; people started to speak more directly; views became better understood; the atmosphere became more open and trusting. The meeting ended. As people left the room, Martin found that he could still hear what they were thinking. “That was the best meeting we?ve ever had.” “All meetings should be like that.” “In future, I?m going to say what I think”. ( )11.It is known from the first paragraph that Martin ________. A(just came back from his business trip in New York B(was found to suffer from a serious heart disease during the trip C(had a good time during his fortnight?s stay in New York D(didn?t like his work in his London office ( )12.Why did Martin hate his company?s team meeting? A(Because it had to be held every Monday morning. B(Because he was tired of hosting such a meeting. C(Because he couldn?t control the meeting that was out of order. D(Because the meeting atmosphere wasn?t open and trusting. ( )13.The underlined part “his newly?acquired talent” in Paragraph 5 means that he could ________. A(still hear the engine noise of the plane though he was back B(clearly hear what the members of the meeting were saying C(actually control the thoughts of the members of the meeting D(express what the meeting members were thinking based on their words and their thoughts ( )14.What do we know about this Monday morning?s meeting from the passage? A(Martin made people say what they were thinking immediately the meeting began. B(Martin was angry at the dullness of the meeting at the beginning of the meeting all the time. C(Martin led people to express their real thoughts with the meeting progressing. D(Many members of the meeting played games and told lies at the meeting all the time. ( )15.It is inferred from the last two paragraphs that towards the end of the meeting Martin would hear such words as “ ________”( A(I was thinking of making a suggestion—but I couldn?t be bothered B(I have got an idea on how we could take this forward. I think we should… C(The usual people say the usual things, so I have no other new ideas D(I could have improved on that idea, but I wasn’t going to say 新闻报道型阅读理解 (一) The crisis at Japan?s Fukushima Dai?Ichi nuclear energy center has raised questions about the future of the nuclear energy industry. Arjun Makhijani is president of the Institute for Energy and Environmental Research in the United States. He says the disaster in Japan is historic. This week, the chairman of America?s nuclear agency said there is little chance that harmful radiation from Japan could reach the United States. Nuclear Regulatory Commission Chairman Gregory Jaczko also said America has a strong program in place to deal with earthquake threats. No new nuclear power centers have been built in the United States since 1979.That was when America?s worst nuclear accident happened at the Three Mile Island center in Pennsylvania. The accident began to turn public opinion against nuclear energy. To support more clean energy production, the Obama government has been seeking billions of dollars in government loan guarantees to build new centers. Presently, about twenty percent of electricity in the United States comes from nuclear energy. But critics say nuclear power is too costly and dangerous to be worth further expansion. German prime minister Angela Merkel said Germany would close all seven nuclear power centers for a moment while energy policy is reconsidered. The European Union is planning to test all centers in its twenty-seven member nations. Developing nations are less willing to slow nuclear expansion. China said it would continue with plans to build about twenty-five new nuclear plants. And India, under a cooperation agreement with the United States, plans to spend billions on new centers in the coming years. Japan has made nuclear energy a national priority since the 1970s. Unlike many major economies, Japan imports eighty percent of its energy. The Nuclear Energy Institute said twenty?nine percent of Japan?s electricity came from nuclear sources in 2011.The government planned to increase that to forty percent by 2017. Nuclear plants supply fourteen percent of global electricity. Nuclear energy is a clean resource, producing no carbon gases. But radioactive waste is a serious unresolved(未解决的) issue. So_is_the_presence_of_nuclear_power_centers_in_earthquake_areas_like_the_one_near_Bushehr ,_Iran. ( )6.Which of the following countries is the least likely to expand its use of nuclear energy? A(China. B(India. C(Iran. D(Germany. ( )7.The government of Japan intended to increase its nuclear energy in 2017 by ________ compared with 2011. A(29% B(40% C(69% D(11% ( )8.Which of the following statements about the use of nuclear power in the U(S. is TRUE? A(Most of the electricity comes from nuclear energy in the U(S. B(The Obama government is against building nuclear power plants. C(A small number of nuclear power plants have been built since 1979. D(A serious nuclear accident happened in America at the end of the 1970s. ( )9.What?s the main idea of this passage? A(The nuclear crisis in Japan has spread to other parts of the world. B(The situation of the world?s nuclear power development after the crisis in Japan. C(Japan?s nuclear disaster will slow the expansion of nuclear plants all over the world. D(Japan?s nuclear disaster won?t affect the nuclear energy production in other countries. ( )10.From the last sentence of the passage, we can see the writer?s attitude towards building a nuclear power plant near Bushehr, Iran is________. A(supportive B(optimistic C(negative D(uncertain (二) Four out of ten women who diet end up heavier than when they started watching their waistline(腰围), a study revealed today. The research also showed that a large percentage of women started noticing the pounds creeping back on just 21 days after reaching their ideal weight. Yesterday, Dr Ian Campbell of the Jenny Craig Weight Management Program said: “In the UK 61.4 per cent of adults are overweight or obese. Successful weight management requires a long-term commitment in order to lose weight successfully and for good. Dieting can be a real challenge so setting realistic goals and remaining focused on them is important. Otherwise as this research shows, women could end up heavier than when they started.” The “Food, Body, Mind” report was publicized by Jenny Craig who quizzed 2,000 women aged between 18 and 65 who diet regularly on their attitudes, beliefs and behaviors around weight loss. Six in ten said they were on a diet then and one in five women said they were on a “continuous diet”( It found the most common triggers to start dieting was seeing their “reflection in the mirror”, preparing for a summer holiday or unflattering photos posted on social networking sites. Other popular reasons include comments by friends or relatives or their other half. However, the study showed that one in ten give up within one day, while almost a fifth manage to make it to a week or more. The average is ten days. Many blamed pressure they put on themselves to lose weight too quickly for the weight gain, which leaves them with a bigger appetite than normal. Others blamed colleagues, who tuck into fatty lunches and snacks unaware of the effect it has on the dieter, while mothers? polishing off their children?s leftovers(剩饭)was another common cause of weight gain. ( )11.Which of the following might be the best title for this article? A(Three reasons to fail in dieting B(Important things for successful diet C(Obesity—a problem for 61.4% adults in the UK D(Four in ten women gain weight on diets ( )12.Women who go on a diet ________. A(are all overweight or obese B(are likely to gain weight again after reaching their ideal weight C(all fail because they are not persistent enough D(all end up heavier than when they start to diet ( )13.The underlined word “triggers” in Paragraph 4 probably means ________. A(effects B(examples C(causes D(imagination ( )14.Which of the following is NOT the reason that many people quit dieting very soon? A(The pressure they put on themselves to lose weight quickly. B(Colleagues who give them fatty lunches and snacks. C(Reflections they see in the mirror. D(Leftovers of children?s taken by their mothers. ( )15.In which column of the newspaper do you probably find the passage? A(Health. B(Economy. C(Sports. D(Education. 广告应用型阅读理解 (一) THE EARTH CENTRE You might have heard about the centre of the earth but have you heard of the Earth Centre? It is a new South Yorkshire attraction—a theme park with a difference. Why does the Earth Centre attract visitors from all over the world? Because it is the world?s first sustainable(可持续的)development park and it aims to promote the idea of sustainable development to all ages. Here you will get to see the world in a way you have never seen it before. ?________________________________________________________________________ There are many impressive attractions. Here are just a few: ?“Nature works” which explore pond life found in the various habitats of the Earth Centre. You don?t just look—if you want you can catch and handle these creatures too. ?The Water Cycle Simulator which allows you to speed around the planet. ?The Living Machine—a full-scale sewage(污水) treatment system for the Earth Centre that uses plants and micro-organisms to create water safe enough to drink. ?Daredevil outdoor activities—if you?re feeling adventurous, try the climbing wall. In addition, there are 400 acres of the Country Park to explore with woods, lakes and a wealth of wildlife. ? Food and drink If you get hungry—no problem! The Centre has its own café serving a range of delicious snacks and meals—all organic of course. ? Shop If you feel like spending some money, you can call at the Earth Shop where you can buy “sustainability products” such as organic cotton T?shirts. ? How to get there By Road—Leave the AIM at Junction 36 and follow the brown Earth Centre signs along the A630, then the A6023 towards Mexborough. The Earth Centre is next right after Conisbrough Railway Station. By Rail—Conisbrough station is next to the entrance to the Earth Centre and is on the Doncaster to Sheffield line. By Boat—The Earth Centre has a wharf(码头) on the River Don. ? Opening Times Open every day (except Christmas Day) 10:00 am—5:00 pm. ( )6.The Earth Centre is popular with a large number of visitors mainly because________. A(it?s convenient to get there B(it?s located in South Yorkshire C(it sells organic cotton T-shirts D(it stresses sustainable development ( )7.Which of the following can be used to fill in the blank in the passage? A(How do you like it? B(What does it offer? C(Who is it intended for, D(What do you hear about it? ( )8.If you are interested in wildlife,you will probably choose________. A(the Living Machine B(the Water Cycle Simulator C(the Daredevil Outdoor Activities D(the Country Park ( )9.From the passage,we can learn that ________. A(the goods at the Earth Shop do no harm to the environment B(micro-organisms probably pollute the clean water C(the food at the Earth Centre is made by hand D(the Earth Centre has no restaurants ( )10.According to the passage, the Earth Centre________. A(is near Conisbrough station B(is open for 8 hours every day C(is busy on Christmas Day D(can be reached by air (二) ※ Health, Wellness and the Politics of Food 9:00—9:45 a(m. Blue Tent Panelists (成员):Jami Bernard, David Kamp, Marion Nestle and Peter Singer. Hosted by Denise Grady, science writer for the New York Times. How does what we eat not only affect our bodies, but also the world,The food and nutrition experts debate the role that the diet plays in both personal and global health, and present a look at food politics. ※Sports Writing:For the Love of the Game 9:50—10:35 a(m. Blue Tent Panelists:Christine Brennan, Ira Rosen, Joe Wallace and Joe Drape. Hosted by William C(Rhoden, sports writer for the New York Times. Whether catching that key moment of victory or defeat, or covering breaking news, sports writers are anything but audience. Listen as some professionals discuss the special experience in reporting of sports news. ※ The Art of the Review 11:15—12:00 a(m. Green Tent Panelists: John Freeman, Barry Gewen, David Orr, Celia McGee and Jennifer Schuessler. Hosted by Sam Tanenhaus, editor for the New York Times Book Review. How much of an effect does the book review have on book sales,Join this group of critics(评论家) as they discuss the reality of book review and best-seller lists, and how they choose books for review. ※New York Writers, New York Stories 3:00—3:45 p(m. Green Tent Panelists: Cindy Adams, Richard Cohen, Ric Klass and Lauren Redniss. Hosted by Clyde Haberman, columnist (专栏作家) for the City Section of the New York Times. Join this inspiring group of New York-centric writers as they talk about why New York is a gold mine of ideas for their work. ( )11.If you are free in the afternoon, you can attend________. A(The Art of the Review B(New York Writers, New York Stories C(Health, Wellness and the Politics of Food D(Sports Writing: For the Love of the Game ( )12.If you like sports writing, you will most probably ________. A(go to Blue Tent at 11:15 a(m B(enjoy Jami Bernard?s talk C(listen to Christine Brennan D(attend the Art of the Review ( )13.Sam Tanenhaus is in charge of ________. A(The Art of the Review B(Health, Wellness and the Politics of Food C(New York Writers, New York Stories D(Sports Writing:For the Love of the Game ( )14.All the four activities above ________. A(are about writing B(will last 45 minutes each C(can be attended freely D(will attract many readers ( )15.We can learn from the text that________. A(sports writers are a type of audience B(the New York Times is popular C(Denise Grady will discuss politics D(book reviews may affect book sales 文化教育型阅读理解 (一) There are many famous museums throughout the world where people can enjoy art. Washington D .C.has the National Gallery of Art (美术馆); Paris has the Louvre; London, the British Museum. Florida International University (FIU) in Miami also shows art for people to see. And it does so without a building, or even a wall for its drawings and paintings. FIU has opened what it says is the first computer art museum in the United States. You don?t have to visit the University to see the art. You just need a computer linked to a telephone. You can call the telephone number of a University computer and connect your own computer to it. All of the art is stored in the school computer. It is computer art, produced electronically (采用电子手段) by artists in their own computers. In only a few minutes, your computer can receive and copy all the pictures and drawings. Robert Shostak is director of the new computer museum. He says he started the museum because computer artists had no place to show their work. A computer artist could only record his pictures electronically and send the records, or floppy discs (软盘), to others to see on their computers. He could also put his pictures on paper. But to print good pictures in paper, the computer artist needed an expensive laser (激光) printer. Robert Shostak says the electronic museum is mostly for art or computer students at schools and universities. Many of the pictures in the museum are made by students. Mr Shostak says the FIU museum will make computer art more fun for computer artists because more people can see it. He says artists will enjoy their work much more if they have an_audience. And the great number of home computers in America could mean a huge audience for the electronic museum. ( )1.The main purpose of this text is to give information about________. A(famous museums through the world B(a computer art museum in Miami, U(S.A. C(art exhibitions in Florida International University D(latest development in computer art ( )2.To see the art in FIU museum, your special needs include________. A(floppy discs B(a computer and a printer C(pictures and drawings on paper D(a computer connected to the museum by telephone line ( )3.What are stored in this museum? A(Paintings drawn by means of computer. B(Different styles of paintings. C(Old paintings. D(Drawings done by art students of FIU. ( )4.The museum was started when________. A(Robert Shostak wanted to do something for computer scientists B(Robert Shostak wanted to help computer artists C(art students needed a place to show their works D(computer scientists wanted to do something about art ( )5.The words “an audience” in the last paragraph here refer to ________. A(art students B(owners of computers C(exhibits in the museum D(those who can enjoy art (二) Diana Jacobs thought her family had a workable plan to pay for college for her 21-year-old twin sons: a combination of savings, income, scholarships, and a modest amount of borrowing. Then her husband lost his job, and the plan fell apart. “I have two kids in college, and I want to say „come home,? but at the same time I want to provide them with a good education,” says Jacobs. The Jacobs family did work out a solution: They asked and received more aid from the schools, and each son increased his borrowing to the maximum amount through the federal loan (贷款) program. They will each graduate with $20,000 of debt, but at least they will be able to finish school. With unemployment rising, financial aid administrators expect to hear from more families like the Jacobses.More students are applying for aid, and more families expect to need student loans. College administrators are concerned that they will not have enough aid money to go around. At the same time, tuition(学费)continues to rise. A report from the National Center for Public Policy and Higher Education found that college tuition and fees increased 439% from 1982 to 2007, while average family income rose just 147%. Student borrowing has more than doubled in the last decade. “If we go on this way for another 25 years, we won?t have an affordable system of higher education,” says Patrick M. Callan, president of the center. “The middle class families have been financing it through debt. They will send kids to college whatever it takes, even if that means a huge amount of debt.” Financial aid administrators have been having a hard time as many companies decide that student loans are not profitable enough and have stopped making them. The good news, however, is that federal loans account for about three quarters of student borrowing, and the government says that money will flow uninterrupted. ( )11.According to Paragraph 1,why did the plan of the Jacobs family fail? A(The twins wasted too much money. B(The father was out of work. C(Their savings ran out. D(The family fell apart. ( )12.How did the Jacobses manage to solve their problem? A. They asked their kids to come home. B. They borrowed $20,000 from the schools. C(They encouraged their twin sons to do part-time jobs. D(They got help from the schools and the federal government. ( )13.Financial aid administrators believe that ________. A. more families will face the same problem as the Jacobses B. the government will receive more letters of complaint C(college tuition fees will double soon D. America?s unemployment will fall ( )14.What can we learn about the middle class families from the text? A. They blamed the government for the tuition increase. B. Their income remained steady in the last decade. C(They will try their best to send kids to college. D. Their debts will be paid off within 25 years. ( )15.According to the last paragraph, the government will ________. A. provide most students with scholarships B. dismiss some financial aid administrators C(stop the companies from making student loans D. go on providing financial support for college students 史地常识型阅读理解 (一) The Best of Alaska Nothing evokes(唤起) Alaska like a whale exploding out of the water or an eagle pulling a silver fish from the river. Combine these images with high mountains, brilliant icebergs and wonderful meals and you really do have the Best of Alaska! Join us for an unforgettable 7-day excursion(远足) to the last frontier! Highlights: JUNEAU: Juneau, the state capital, is rich in culture and scenic beauty. It is here that we start and end our trip. HAINES: Haines is a small community located along the fjords(海湾). The natural beauty and expansive wilderness found here have made Haines a premier center for adventure in Alaska. ALASKA INDIAN ARTS: Alaska Indian Arts is a nonprofit corporation dedicated to the preservation and continuation of traditional native craft and culture of the Northwest Native Tribes. We spend a few hours learning carving, native beading(玻璃珠) and culture from these master artists. SKAGWAY: Skagway is a lively town, which still reflects its gold rush roots and contains colorful shops. In Skagway, we stop by the Klondike Gold Rush National Park Visitor?s Center and ride the White Pass Yukon Route Railway. GUSTAVUS: Gustavus is the gateway to Glacier Bay National Park. We?ll stay at a comfortable lodge here for two nights. This will be the base for both the whale-watching excursion and a full day cruise in Glacier Bay. DATES/PRICES: 2012: May 15, June 17, July 16, August 14. 7 days—$3500, including lodging, all meals, excursions, guides, park fees, sales taxes, and transportation between Juneau, Skagway, Haines, and Gustavus. Not included: Alcohol, personal items, airfare to and from Juneau. Contact: E-mail: info@alaskamountainguides.com Call: 800,766,3396 Write: Alaska Mountain Guides & Climbing School P(O. Box 1081, Haines AK 99827 ( )6.You can feel the history of the local place at ________. A(Juneau B(Skagway C(Haines D(Gustavus ( )7.You can watch a whale exploding out of the water at ________. A(Glacier Bay National Park B(the Klondike Gold Rush National Park C(the White Pass Yukon D(the fjords ( )8.According to the advertisement, the participants of the excursion ________. A(will spend one day experiencing the local culture at Alaska Indian Arts B(can contact Alaska Mountain Guides & Climbing School by email or fax C(are able to experience some adventurous activities in Haines D(can get lots of traditional native artworks free of charge ( )9.Which of the following is a participant required to pay additional fees? A(Going from Gustavus to Juneau by train. B(A full day cruise in Glacier Bay. C(Having the last supper at Juneau. D(Flying from Juneau to his hometown. ( )10.It is known from the advertisement that ________. A(the native culture of the Northwest Native Tribes is dying out B(Juneau, as the state capital, is much more beautiful than Haines C(Skagway was the richest town of Alaska in the past D(Alaska is famous for its culture, history and natural beauty (二) People have been acting like people—in other words, they?ve been making tools, creating ceremonies, and sharing food—for a long time. That?s the conclusion of a recent study from South Africa?s southern coast. There, in a cave lying above the sea, researchers from Arizona State University have found evidence that humans were behaving in surprisingly complex ways as early as 164,000 years ago. Our species, Homo sapiens, appeared an estimated 200,000 years ago. The cave held three important clues about the behavior of these Stone Age people. First, the researchers found the remains of all kinds of shellfish. The people who lived in the cave probably collected these creatures from rocky shores and tide pools and brought them to the cave to eat. The researchers propose the early Africans moved to the South African coast between 195,000 and 130,000 years ago. Around that time, the climate inland turned relatively cold and dry. Therefore, there were fewer plants and animals to eat away from the coast. When these ancient people moved to the coast, they probably experienced a major cultural shift, the researchers suspect. That?s because observations of modern hunter?gatherer societies suggest that men are more likely to hunt for big animals when people live inland. On the coast, women play a more important role in providing food by gathering plants and shellfish. As for the second clue, the researchers unearthed 57 pieces of reddish pigment(颜料). The researchers think that the cave habitants used the paint for coloring their bodies or for other ceremonies. Symbolic behavior is a clearly human feature. Finally, the search discovered over 1,800 stone tools, including well-crafted blades(刀片). These blades came in various sizes. The smallest were just less than a half-inch wide. Ancient people may have attached these blades to the end of a stick to make spears or other tools. ( )11.What is the passage mainly about? A(Ancient cave behavior. B(New evidence of ancient life. C(The history of human beings. D(Human behavior?s development. ( )12.The early Africans moved to the South African coast with the main purpose of ________. A(avoiding badly cold and dry weather B(seeking the cultural development C(seeking enough food to live on D(living a newer and better life ( )13.According to the passage, the cave habitants________. A(knew how to use colors for decoration B(made a living largely by hunting animals C(only could made tools with rough blades D(could hunt for fish in the sea with ships ( )14.After the early Africans moved to the coast, what probably happened to their culture? A(Women?s role was as important as men?s role in families. B(They were no longer used to living inlands but living on the coast. C(Men?s role was more important than women?s role in earning food. D(Women?s role was more important than men?s role in earning food. ( )15.The new discoveries in the cave should support the theory that ________. A(modern human behavior developed smoothly fast from the ancient time B(modern human behavior developed slowly from the ancient time C(South Africa is the birthplace of the earliest human beings in the world D(modern human behavior is largely the same as ancient human behavior 社会生活型阅读理解(一) (一) School meals are junk and don?t provide the nutrition a body needs to grow, says Jamie Oliver, and he decides to do something about it. Schoolchildren in France who eat school dinners are likely to have a well-balanced, four-course meal with fresh ingredients (成分) costing up to ,1.10 to make. But British schoolchildren are fed mainly cheap-processed meat, frozen pizzas and potato smiley faces, costing on average just 37 pence per meal. In fact the money spent on school dinners is so small that schools can afford only whatsome people describe as junk food rather than fresh natural ingredients. This means that their meals don?t provide the nutrients and goodness children?s bodies need to develop properly. No wonder there?s growing concern about the health of the nation?s schoolchildren. Celebrity chef Jamie Oliver was so concerned about school dinners that he decided to try to improve them by teaming up with the kitchen staff at Kidbrooke School in Grenwich. He took a lot of time to learn how to get the kids to stop eating the junk and move to healthy food. This is not easy! But it is possible with some hard work and determination! After a lot of hard work from both Jamie and the school?s cooks, Kidbrooke is now serving some of the best school food in Britain. The menu includes fresh butcher?s sausages with creamy mash and onion gravy, chilli corn with basmati rice and fresh fruit salad. Incredibly, Jamie and the staff managed to create this healthier menu with the same 37 pence budget. But the school chefs admit cooking healthy food isn?t as easy as simply putting burgers and chips in the oven. Then came the good news! The government has promised to spend ,280 million to improve school dinners across the country. Well done Jamie! This means that the national average cost will rise from 37p to 50p in primary schools and from 37p to 60p in secondary schools. ( )1.What does the author try to tell us in the first paragraph? A(School meals in France are the most expensive in the world. B(British schools don?t provide the students with healthy food. C(School meals in Britain are cheaper than those in France. D(British school meals include many kinds of healthy food. ( )2.British schools provide students with such meals because________. A(most of the school children are too fat B(people show little concern about the health of schoolchildren C(schools don?t have enough money D(schools don?t want to spend much money ( )3.From the passage we can see that________. A(the British government ignores schoolchildren?s health B(Jamie creates a healthy menu with the help of the government C(schoolchildren in Britain will have the best meals D(cooking healthy food is not an easy job ( )4.The good news for the British is that________. A(schoolchildren will have the best food in Britain B(the menu Jamie Oliver created is the best C(parents have begun to pay attention to children?s food at school D(the government has begun to do something about the fact ( )5.What is the best title for this newspaper story? A(A Healthier Menu for School Dinners B(Schoolchildren in Britain Can?t Develop Properly C(Dinners Lacking Nutrients in School D(Schools with the Best Food (二) Life comes in a package. This package includes happiness and sorrow, failure and success, hope and despair. Life is a learning process. Experiences in life teach us new lessons and make us a better person. With each passing day we learn to handle various situations. Love Love plays a pivotal role in our life. Love makes you feel wanted. Without love a person could go hayward(不归路) and also become cruel and ferocious. In the early stage of our life, our parents are the ones who show us with unconditional love and care; they teach us about what is right and wrong, good and bad. But we always tend to take this for granted. It is only after marriage and having kids that a person understands and becomes sensitive to others? feelings. Kids make a person responsible and mature and help us to understand life better. Happiness and Sorrow Materialistic happiness is short?lived, but happiness achieved by bringing a smile on others? face gives a certain level of fulfillment. Peace of mind is the main link to happiness. No mind is happy without peace. We realize the true worth of happiness when we are in sorrow. Sorrow is basically due to death of a loved one, failure and despair. But these things are temporary and pass away. Failure and Success Failure is the path to success. It helps us to touch the sky, teaches us to survive and shows us a specific way. Success brings in money, fame, pride and self-respect. Here it becomes very important to keep our head on our shoulder. The only way to show our gratitude to God for bestowing success on us is by being humble, modest, courteous and respectful to the less fortunate ones. Hope and Despair Hope is what keeps life going. Parents always hope their children will do well. Hope makes us dream. Hope builds in patience. Life teaches us not to despair even in the darkest hour, because after every night there is a day. Nothing remains the same. We have only one choice keep moving on in life and be hopeful. Life teaches us not to regret over yesterday, for it has passed and is beyond our control. Tomorrow is unknown, for it could either be bright or dull. So_the_only_alternative_is_to_work_hard_today,_so_that_we_will_enjoy_a_better_tomorrow. ( )11.In the passage, the author compares life to ________. A(a package B(joy and sorrow C(failure and success D(hope and despair ( )12.In which section can readers probably read the passage? A(Opinions. B(Politics. C(Education. D(Lifestyles. ( )13.How is the passage organized? A(In order of rules. B(In order of frequency. C(By making comparison. D(In order of importance. ( )14.What?s the general meaning of the last sentence of the whole passage? A(If you work hard, you?ll have a better life in the future. B(When choosing between “today” and “tomorrow”, you?d better enjoy tomorrow. C(What you only can choose is to work hard today rather than tomorrow. D(Enjoying a better tomorrow is your only choice. ( )15.What?s the purpose of the passage? A(To encourage people to love each other. B(To tell people how to deal with happiness and sorrow. C(To remind people of keeping having a beautiful dream. D(To give readers some information about life. 社会生活型阅读理解(二) (一) Someday a stranger will read your e-mail without your permission or scan the websites you?ve visited.Or perhaps someone will casually glance through your credit card purchases or cell phone bills to find out your shopping preferences or calling habits. In fact,it?s likely that some of these things have already happened to you.Who would watch you without your permission? It might be a girlfriend,a marketing company, a boss,a policeman or a criminal.Whoever it is,they will see you in a way you never intended to be seen—the 21st century equal to being caught naked(裸露的)( Psychologists tell us boundaries are healthy, though it?s important to reveal(透露)yourself to friends,family and lovers in stages,at appropriate times.Actually few boundaries remain.The digital breadcrumbs(面包屑)you leave everywhere make it easy for strangers to reconstruct who you are,where you are and what you like.In some cases,a simple Google search can reveal what you think.Like it or not,increasingly we_live_in_a_world_where_you_simply_cannot_keep_a_secret. The key question is:Does that matter? For many Americans, the answer apparently is“no”(When opinion polls ask Americans about privacy, most say they are concerned about losing it. A survey found a majority of people are pessimistic about privacy, with 60 percent of respondents saying they feel their privacy is“slipping away, and that bothers me”( But people say one thing and do another.Only a small part of Americans change any behavior in an effort to preserve their privacy.Few people turn down a discount at tollbooths(收费亭)to avoid using the EZ-Pass system that contracts(跟踪) automobile movements.And few turn down supermarket loyalty cards.Privacy economist Alessandro Acquits has run a series of tests that reveal people will submit personal information like Social Security numbers just to get their hands on a pitiful 50-cent-off coupon(优惠券)( But privacy does matter—at least sometimes.It?s like health: when you have it,you don?t notice it.Only when it?s gone do you wish you?d done more to protect it.[ ( )1.From Paragraph 2,we can infer________. A(criminals are easily caught on the spot with advanced technology B(people tend to be more frank with each other in the information age C(in the 21st century people try every means to look into others? secrets D(people?s personal information is easily accessed without their knowledge ( )2.What would psychologists advise on the relationships between friends? A(There should be a distance even between friends. B(There should be fewer quarrels between friends. C(Friends should always be faithful to each other. D(Friends should open their hearts to each other. ( )3.Why does the author say“we live in a world where you simply cannot keep a secret”, A(There are always people who are curious about other?s affairs. B(Many search engines profit by revealing people?s identities. C(People leave traces around when using modern technology. D(Modern society has finally developed into an open society. ( )4.What do most Americans do with regard to privacy protection? A(They change behavior that might disclose their identity. B(They talk a lot but hardly do anything about it. C(They rely more and more on electronic equipment. D(They use various loyalty cards for business deals. ( )5.According to the passage,privacy is like health in that ________. A(its importance is rarely understood B(it is something that can easily be lost C(people will make every effort to keep it D(people don?t treasure it until they lose it (二) Some years ago industries had more freedom than they have now,and they did not need to be as careful as they must today. They did not need to worry a lot about the safety of the new products that they developed. They did not have to pay much attention to the health and safety of the people who worked for them. Often new products were dangerous for the people who used them;often conditions in the work place had very bad effects on the health of the workers. Of course sometimes there were real disasters(灾难) which attracted the attention of governments and which showed the need for changes.Also scientists who were doing research into the health of workers sometimes produced information which governments could not ignore.At such times, there were inquiries into the causes of the disasters or the problems. New safety rules were often introduced as a result of these inquiries;however, the new rules came too late to protect the people who died or who became seriously ill. Today many governments have special departments which protect customers and workers. In the U. S(, for example, there is a department which tests new airplanes and gives warnings about possible problems. It also makes the rules that aircraft producers must follow. Another department controls the foods and drugs that companies sell.A third department looks at the places where people work,and then reports any company that is breaking the laws which protect the health and safety of workers. Of course,new government departments and new laws cannot prevent every accident or illness,but they are having some good results. Our work places are safer and cleaner than before. The planes and cars which we use for travel are better. Producers are thinking more about the safety and health of the people who buy and use their products. ( )11.The main topic of the passage is________. A(conditions in the work place B(the freedom of industries in the past C(changes in industrial production D(the safety and health of workers and customers ( )12.It can be inferred from the passage that in the past________. A(workers often got ill because of the poor working conditions B(companies were free to put out any products they wanted to C(many people were killed by dangerous products D(industries were as careful in management as they are today ( )13.It is implied in the passage that________. A(governments and companies had different opinions about the safety of products B(governments paid little attention to the safety of products C(government officials often did not listen to scientists D(in the past no safety laws were introduced by governments ( )14.Some years ago safety rules________. A(were put forward due to scientists? recommendations B(came into being as a result of the workers? demands C(were introduced because quite a number of people were killed or seriously injured D(were effective enough to protect workers and customers ( )15.The special departments protect customers and workers in many ways EXCEPT by________. A(testing new products B(controlling the sale of products C(designing new products D(inspecting work places 科普知识型阅读理解(一) (一) When top engineers in Germany wanted to build a more energy-efficient car, they headed to the natural history museum to study dolphins and sharks. But it was the boxfish (盒子鱼)that interested them. “We were surprised when this clumsy-looking fish became our model for designing an aerodynamic (空气动力学的)car”,says Thomas Weber. He is the research and development chief for the car company Daimler. An aerodynamic design reduces wind drag and increases fuel efficiency. The boxfish may look clumsy, but it can start, stop,back up and zigzag (曲折前 进)through the water with ease. And it does all this using surprisingly little energy. Daimler?s bionic (仿生学的)car is modeled on the fish?s boxy skeletal (骨骼的) system. Like the fish, the car is fast and drives easily. It?s efficient too. The science behind nature-inspired inventions is called biomimicry (生物仿生). Biologist Janine Benyus came up with the term. As co-founder of the Biomimicry Guild, she has worked with cereal companies,sneaker designers and others to develop products based on Nature?s best ideas. This month,the Nature?s 100 Best List will be revealed at the World Conservation Congress (代表大会)in Barcelona,Spain. Benyus co-authored the list of nature-inspired designs with economist Gunter Pauli. In Zimbabwe,a country in southern Africa,engineers designed energy-efficient buildings modeled on termite (白蚁)mounds (土堆). The buildings use vents (孔) to keep the air flowing and the temperature cool. A company in Atlanta,US,developed a self-cleaning paint modeled on the lotus (荷 花)leaf. When the paint dries,it becomes bumpy (不平的),just like the lotus leaf. Rain drops form on the bumps and roll off,carrying dirt along with them. Nature has done billions of years of research. Only the best-designed products have survived. Companies owe Mother Nature a debt for all that hard work,says Benyus. For scientists and inventors, the possibilities for copying nature are limitless. Technology may never be as efficient as nature, or as colorful. But that doesn?t mean we should stop trying to learn from the world around us. Take a walk outside. Watch a squirrel climb up a tree. Look closely at the veins (纹理)of a leaf. Hear the birds chirping in the trees. Such small details may help solve big problems in the future. ( )6.Which is TRUE according to the passage? A(In America a self-cleaning paint was modeled on the lotus flower. B(Zimbabwe engineers learned from termites to create highly advanced heating systems. C(After the boxfish,many other ideas of new products are borrowed from nature. D(Nature is a good source to turn to for improving technology. ( )7.The clumsy-looking boxfish interested the scientists because of the following reasons EXCEPT that________. A( it swims using little energy B( being boxy, it looks quite like a car C( some engineers had researched on how to swim easily in the water D( it is a speedy swimmer ( )8.Why did top engineers in Germany go to the natural history museum? A( They wanted to pay a visit to the animals there because they were so lovely. B( They wanted to carry out a research of boxfish and dolphins. C( They wanted to search for a model to base on for building a more-efficient car. D( They wanted to study the science of survival. ( )9.We can infer all of the following from the passage EXCEPT ________. A( it is believed that the creation of the airplane might have originated from the bird flying in the sky B( the atmosphere on the earth cannot be transferred into the original idea C( dolphins and sharks may become the source of scientists,inventions or creations D( if you observe and explore nature,you may make great discoveries or become great inventors ( )10.What?s the main idea of the passage? A( How Daimler?s bionic car came into being. B( Nature has inspired a lot of inventions and will continue to. C( Scientists and inventors? contribution to modern industry comes from nature. D( The Nature?s 100 Best List. (二) If cars had wings,they could fly and that just might happen, beginning in 2012.The company Terrafugia, based in Woburn, Massachusetts, says it plans to deliver its car-plane, the Transition, to customers by the end of 2012. “It?s the next „wow? vehicle,”said Terrafugia vice president Richard Gersh.“Anybody can buy a Ferrari, but as we say, Ferraris don?t fly.” The car plane has wings that unfold for flying—a process the company says takes one minute—and fold back up for driving. A runway is still required to take off and land. The Transition is being marketed more as a plane that drives than a car that flies, although it is both. The company has been working with FAA to meet aircraft regulations, and with the National Highway Traffic Safety Administration to meet vehicle safety regulations. The company is aiming to sell the Transition to private pilots as a more convenient and cheaper way to fly. They say it saves you the trouble from trying to find another mode of transportation to get to and from airports: You drive the car to the airport and then you?re good to go. When you land, you fold up the wings and hit the road. There are no expensive parking fees because you don?t have to store it at an airport—you park it in the garage at home. The car-plane is designed to fly primarily under 10,000 feet. It has a maximum takeoff weight of 1,430 pounds, including fuel and passengers. Terrafugia says the Transition reduces the potential for an accident by allowing pilots to drive under bad weather instead of flying into marginal(临界)conditions. The Transition?s price tag: $194,000. But there may be additional charges for options like a radio, transponder or GPS. Another option is a full-plane parachute. “If you get into a very awful situation, it is the necessary safety option,” Gersh said. So far, the company has more than 70 orders with deposits. “We?re working very closely with them, but there are still some remaining steps,” Brown said. ( )11.We can learn from the first paragraph that________. A(car-planes will be popular in 2012 B(people might drive a car-plane in 2012 C(both Transition and Ferrari can take off and land D(Richard Gersh is the vice president of Massachusetts ( )12.It takes the car-plane one minute to________. A(fold and unfold its wings B(unfold wings for flying C(land in the airport D(meet flying safety regulations ( )13.According to the passage, which of the following is NOT true? A(The car-plane needs a runway to take off and land. B(To meet aircraft regulations, the company has been working with FAA. C(The car-plane may fly as high as normal planes. D(People can park the car-plane in the garage at their home. ( )14.The underlined word “it” in the last but one paragraph refers to________. A(the radio B(the transponder C(the GPS D(the full-plane parachute ( )15.What?s the best title for the passage? A(Cars With Wings May Be Just Around The Corner B(Which To Choose: A Ferrari Or A Car-Plane? C(A More Convenient And Cheaper Way To Fly D(Cars With Wings Can Fly As Fast As Plane 科普知识型阅读理解(二) (一) Plants have family values, too, it seems, with new research suggesting they can recognize close relatives in order to work together. An ability to tell family from strangers is well known in animals, allowing them to cooperate and share resources, but plants may possess similar social skills, scientists believe. Susan Dudley and Amanda File of McMaster University in Ontario, Canada, report they have demonstrated for the first time that plants can recognize their kin. This suggests that plants, though lacking recognition and memory, are capable of complex social interactions. “Plants have this kind of hidden but complicated social life,” Dudley said. The study found plants from the same species of beach-dwelling wildflower grew aggressively alongside unrelated neighbors but were less competitive when they shared soil with their families. Sea rocket, a North American species, showed stronger and healthier root growth when planted in pots with strangers than when raised with relatives from the same maternal(母系的) family, the study found. This is an example of kin selection, a behavior common in animals in which closely related individuals take a group approach to succeeding in their environment, the researchers said. Kin selection also applies to competition, because if family members compete less with each other, the group will do better overall. “Everywhere you look, plants are growing right up next to other plants,” Dudley said,“ Usually it?s a case of each plant for itself. But sometimes those plants are related, and there are benefits to not wasting resources on being competitive, and there is not really a cost to not being competitive as long as your neighbor is also not being competitive.” Learning and memory appear to be important for kin recognition in animals, but this isn?t an option for plants, she noted. Some researchers speculate(猜测) that plants communicate through their roots, identifying themselves using tiny chemical signatures specific to each plant?s family. ( )6.What?s the main idea of the message? A(Studies find plants can recognize, communicate with relatives. B(Kin selection is important for plants. C(Animals can recognize and memorize their relatives. D(Competition asks plants to recognize their relatives. ( )7.Which of the following is NOT right about animals? social skill? A(Animals can recognize and memorize their relatives. B(Animals? social skill is to cooperate and share resources. C(Animals? social skill can recognize close relatives in order to work together. D(Animals? social skill is no use at all. ( )8.Plants? kin selection is to ________. A(grow well B(compete with other kinds of plants C(strengthen the relationship among siblings D(find which one is the best ( )9.From the passage,we learn that ________. A(sea rocket is a South American species B(sea rocket grows aggressively alongside unrelated neighbors C(sea rocket grows aggressively alongside its siblings D(sea rocket is a kind of bush without flowers ( )10.How can the plants communicate with each other according to experts? suppose? A(Plants communicate by using tiny chemical signatures specific to each plant?s family. B(Plants communicate with each other through their roots. C(Plants communicate with each other by their leaves. D(Plants communicate with each other with their flowers. (二) Schools are almost all connected to the Internet. But some have more technology, and use it more, than others. For example, some schools use computers for activities like video conferencing, to bring the world into the classroom. And some classrooms are equipped with things like a Smart Board, a kind of interactive whiteboard. Interactive whiteboards are large displays for presentations. They connect to a computer and can operate by touch. They can be used for documents or writing or to project video. Some teachers are trying creative new ways to teach with devices like iPods and mobile phones. But educators say the most important thing, as always, is the content. Yet technology can have special importance in some cases. Cosmobot is a therapy robot. It stands about half a meter tall and has a blue body and a friendly face with big eyes. One child who works with it is six-year-old Kevin Fitzgerald. Kevin has developmental dyspraxia;_he has difficulty moving his mouth and tongue. He works with Carole Semango-Sprouse as he interacts with the Cosmobot during therapy for his condition. Here, he uses a set of buttons attached to a computer to make the silent robot move forward, backward or around in circles. Kevin?s mother thinks the robot has had a calming influence, helping her son get along better with his friends. Cosmobot was developed by Anthro Tronix. Corinna Lathan started the company ten years ago to work with children with cerebral palsy, Down?s syndrome, autism and other developmental disabilities. Children become friends with the robot, she says. That can have a big effect on their behavior, helping them work harder and longer in therapy sessions. Corinna Lathan is currently working with a British company to develop other socially helpful robots. She says they are still considered research tools in the United States, and not used as much as in places like Britain and Japan. But she hopes to change that. ( )11.It?s known from the text that ________. A(schools have easy access to the Internet today B(new technology is seldom applied to teaching C(the Internet is of equal importance in schools D(using computers for activities is labor-saving ( )12.Which of the following is NOT right about Smart Board? A(It?s a kind of teaching facility. B(It can be used to project video. C(It?s a display for presentation. D(It?s not easy to be connected to computers. ( )13.What?s “dyspraxia” according to the text? A(Speech barrier. B(Movement disorder. C(Visual impairment. D(Loss of memory. ( )14.Which of the following is RIGHT according to the text? A(Cosmobot is designed to play with. B(Cosmobot is costly and not user-friendly. C(Cosmobot can be controlled by a computer. D(Anthro Tronix is a designer of a certain company. ( )15.What can we learn from Corinna Lathan? A(She?s employed by a British company. B(Cosmobot is popular around the world. C(She wants to make previous products widespread. D(IPods and mobile phones are valued in class.
/
本文档为【学生用 英语阅读理解四大题型专讲专练】,请使用软件OFFICE或WPS软件打开。作品中的文字与图均可以修改和编辑, 图片更改请在作品中右键图片并更换,文字修改请直接点击文字进行修改,也可以新增和删除文档中的内容。
[版权声明] 本站所有资料为用户分享产生,若发现您的权利被侵害,请联系客服邮件isharekefu@iask.cn,我们尽快处理。 本作品所展示的图片、画像、字体、音乐的版权可能需版权方额外授权,请谨慎使用。 网站提供的党政主题相关内容(国旗、国徽、党徽..)目的在于配合国家政策宣传,仅限个人学习分享使用,禁止用于任何广告和商用目的。

历史搜索

    清空历史搜索